TOP-12 самых аэродинамически совершенных авто
Уже все в курсе, какое свойство кузова влияет на почти каждый аспект автомобиля? Заметно улучшает экономию топлива, особенно на больших скоростях. Уменьшает разгон до сотни (пусть хоть и всего на доли секунды). Даже влияет на устойчивость на прямой и в скоростных поворотах? Это рожденная в недрах аэродинамической трубы аэродинамика.
Идеальное аэродинамическое тело – капля воды, летящая к земле. Вот почему многие футуристические концепт-кары, которые подчеркивают важность аэродинамики, похожи на кусок желе, шлепнутого о стену. В них пытаются натянуть форму капли на узлы и агрегаты автомобиля и придать ей привлекательный вид. Но в серию такие машины не идут. Производители считают, что средний потребитель не заинтересован проводить дорогу на работу в потусторонней колеснице. На данный момент, чтобы продать автомобиль, по-прежнему необходимо, чтобы он выглядел как старый привычный автомобиль.
Конструкторы идут на компромисс, и он дается им с большим трудом.
Они не слишком меняют форму автомобиля, но делают все возможное, чтобы снизить сопротивление кузова воздуху. 15 лет назад Opel Calibra установил непостижимый для того времени результат – коэффициент сопротивления (Cd) равный 0,26. И сегодня Cd равный 0,26 годится только для 10-го результата. Вот 12 самых аэродинамических транспортных средств, которые вы можете купить прямо сейчас:
12. Audi A6: 2011 (Cd 0.26)
Вы можете утверждать, что A6 ничуть не отличается от других Audi. Ан нет! Именно у седана A6 лучший коэффициент лобового сопротивления 0,26. Даже у A7 Sportback хуже. В R8 потоки воздуха организуют дополнительную прижимную силу, и Cd далек от значения 0,26
11. BMW i8: 2014 (Cd 0.26)
С нуля до 100 км/ч i8 разгоняется менее чем за 4,5 секунды. Он легкий, с низкой посадкой – несомненно, автомобиль для водителя. Но кроме того, он гибрид, и расход топлива для него – критически важное значение. Отличная аэродинамика – необходимая составляющая для достижения вышеуказанных целей.
Для воздуха на кузове и днище организованы специальные протоки, щели и бороздки. Круть!
10. Mazda3 Sedan: 2012 (Cd 0.26)
Главный трюк маленькой Mazda – активные жалюзи решетки, установленные в переднем бампере. Они автоматически закрываются, когда двигатель не требует охлаждения, и отправляют воздушные потоки вдоль кузова. Система не уникальна, но чаще используется на очень крутых тачках. Так вот, на поле аэродинамики Mazda 3 играет с ними на равных.
9. Mercedes-Benz B-Class: 2012 (Cd 0.26)
Мы не ожидали, что в этом списке окажется минивэн. Но вот, пожалуйста, пухлый B-Class имеет прекрасный Cd. Инженеры Mercedes провели около 1100 часов в аэродинамической трубе за оптимизацией каждой поверхности и каждой линии B-Class, даровав ему удивительную способность разрезать воздух.
8. Nissan GT-R, 2011 (Cd 0.26)
Очень удивительный результат, учитывая, сколько прижимной силы нужно GT-R, чтобы оставаться в контакте с асфальтом.
“Аэролезвия” по краям крыльев обеспечивают оптимальный воздушный поток вокруг шин и вдоль кузова, в то время как дизайн переднего бампера и заднего диффузора делает его еще более обтекаемым. Безусловно, самый быстрый автомобиль в этом списке.
7. Peugeot 508, 2011 (Cd 0.25)
Peugeot демонстрирует вполне обтекаемый силуэт и, как следствие, низкий Cd. Здесь нет никакой сверхъестественной магии – просто правильные формы (и гений инженеров).
6. Hyundai Sonata Hybrid, 2013 (Cd 0.25)
Гибридная версия семейного седана Hyundai довольно сильно отличается от своих стандартных братьев и сестер. Передние и задние бамперы имеют глубокие направляющие для воздуха, на боковинах добавили аэродинамические юбки и даже специально спроектированные 17-дюймовые диски, которые помогают уменьшить лобовое сопротивление. Все очень важно, когда основной целью определено: как можно дальше уехать на одном баке.
5. Toyota Prius, 2010 (Cd 0.
25)
А вы думаете, почему все поколения Toyota Prius, начиная с 90-х годов, такие странные? Можно любить или ненавидеть этот дизайн, но нет никаких сомнений в его аэродинамической эффективности. В современном Prius кузов венчает тоненький спойлер, который вместе с другими элементами задка уменьшает турбулентные завихрения до минимума. Самый известный в мире гибрид также имеет особенно крошечные зазоры между панелями кузова и особенно точную подгонку остекления. Мелочей нет.
4. Mercedes-Benz S-Class, 2014 (Cd 0.24)
S-Class всегда в авангарде инноваций, так что не удивительно, что флагман Mercedes-Benz один из самых аэродинамически совершенных автомобилей в мире. Совершенствуя аэродинамику, инженеры гнались за снижением шума. Великолепный, выверенный кузов плюс автоматическое опускание подвески на скоростях свыше 120 км/ч.
3. Tesla Model S: 2012 (Cd 0.24)
Полностью электрическая Tesla напичкана новыми технологиями.
Это относится и к аэродинамике. У нее “активные” дверные ручки, которые прячутся в кузов при движении и тем самым не создают лишнего сопротивления набегающему потоку воздуха. Даже когда в крыше открыт панорамный люк, перед ним выставляется маленький экран, чтобы не только минимизировать звуковое давление в салоне, но и оптимизировать поток воздуха.
2. Mercedes-Benz CLA: 2013 (Cd 0.22)
CLA является самым убедительным доказательством того, что автомобиль может быть визуально привлекательным и очень обтекаемым одновременно. На CLA установлены специально спрофилированные по воздушному потоку передние стойки и боковые зеркала, улучшена аэродинамика дисков колес, и выштамповки на кузове специально выправляют воздушные потоки. Даже глушитель был разработан с учетом воздушных потоков. И так в каждой детали.
1. Volkswagen XL1: 2013 (Cd 0.19)
А вот эта модель не приемлет компромиссов – она была построена не в угоду консерваторам-покупателям, а согласно последним достижениям аэродинамики.
XL1 мало похож на автомобиль, и то, что его пустили в серию (если можно назвать серией план на изготовление 250 штук), можно считать чудом. Здесь кузов сильно сужается к задку, чтобы имитировать обтекаемую форму дельфина. Задние колеса закрыты аэродинамическими щитами, а вместо больших боковых зеркал заднего вида стоят крохотные камеры. Все воздухозаборники могут закрываться, и шины почти такие же тощие, как у горного велосипеда. Все это помогает XL1 показать впечатляюще низкий коэффициент аэродинамического сопротивления 0,19. У нас есть победитель!
Источник: topgearrussia.ru
Якщо ви знайшли помилку, будь ласка, виділіть фрагмент тексту та натисніть Ctrl+Enter.
Подобається контент? Підтримай Autogeek на Patreon!
Коэффициент аэродинамического сопротивления автомобилей таблица
В процессе проектирования и создания конструкторами очень тщательно прорабатывается аэродинамика автомобиля, поскольку она оказывает значительное влияние на технические показатели модели.
При движении автомобиля большая часть мощности силовой установки уходит на преодоление сопротивления, создаваемого воздухом. И правильно созданная аэродинамика автомобиля позволяет уменьшить это сопротивление, а значит на борьбу с противодействием находящего воздушного потока потребуется затратить меньше мощности, и соответственно – топлива.
Измерение аэродинамики автомобиля проводится для изучения сил, создаваемых воздушным потоком и воздействующих на транспортное средство. И таких сил несколько – подъемные и боковые, а также лобовое сопротивление.
Аэродинамическое сопротивление автомобиля
В процессе проектирования и создания конструкторами очень тщательно прорабатывается аэродинамика автомобиля, поскольку она оказывает значительное влияние на технические показатели модели.
При движении автомобиля большая часть мощности силовой установки уходит на преодоление сопротивления, создаваемого воздухом. И правильно созданная аэродинамика автомобиля позволяет уменьшить это сопротивление, а значит на борьбу с противодействием находящего воздушного потока потребуется затратить меньше мощности, и соответственно – топлива.
Измерение аэродинамики автомобиля проводится для изучения сил, создаваемых воздушным потоком и воздействующих на транспортное средство. И таких сил несколько – подъемные и боковые, а также лобовое сопротивление.
EV1 General Motors
Хотя компания General Motors официально и не продавала свою модель EV1, а только сдавала в аренду, этот автомобиль написал в автопромышленности свою историю. Этот автомобиль вместил в себя как и разочарования (проект был сырой, и машина была ненадежна), так и позитив. Этот автомобиль, начиная с 1996 года, стал первым электромобилем в автопромышленности. Всего было произведено 1000 автомобилей.
Смотрите также: 11 GIF фото которые демонстрируют как работает аэродинамика в автомобилях
Машина оснащалась простыми свинцовыми или никель-металлогидридными батареями. Но, несмотря на это, запас хода у электрического транспортного средства был потрясающим – 230 км. И все это благодаря конструкции кузова, который имел невероятный коэффициент сопротивления воздуха, составляющий всего 0,19 cd.
Лобовое сопротивление и коэффициент Сх
По большей части все работы с кузовом авто направлены на преодоление лобового сопротивления, поскольку именно эта сила самая значительная.
Движение потоков воздуха
За основу при расчетах берется сила сопротивления воздуха. Для вычисления результата используются такие данные как плотность воздуха, площадь поперечной проекции авто, коэффициент аэродинамического сопротивления (Сх) — это важнейший показатель в аэродинамике автомобиля. При этом на силу сопротивления в значительной мере влияет также скорость движения. Так, увеличение скорости вдвое будет сопровождаться повышением сопротивлением в 4 раза. Скорость один из мощных факторов увеличения расхода.
Например, для хорошо обтекаемого авто с площадью проекции 2 м 2 и коэффициентом 0,3 при движении на скорости 60 км/ч для преодоления сопротивления воздуха необходимо 2,4 л.с., а при скорости 120 км/ч уже 19,1 л.с. Разница расхода топлива при таких условиях достигает 30% на 100 км.
Если вам, в данный момент, требуется максимальная экономия топлива, необходимо придерживаться постоянной скорости около 60 км/ч. В этом режиме движения расход будет минимальным даже у авто с большим Cx.
Рассмотрим все по-простому. У воздуха есть своя плотность, причем немалая. При движении автомобилю приходится проходить через имеющиеся воздушные массы, при этом создается поток, который обтекает кузов. И чем легче авто будет «резать» воздушную массу, тем меньше он затратит на это энергии.
Но не все так просто. Во время движения перед авто создается область увеличенного давления (машина сжимает воздушную массу), то есть спереди образуется такой себе невидимый барьер, осложняющий «разрезание» воздушной массы.
Также после обтекания кузова происходит отрыв воздушного потока от поверхности, что становиться причиной появления завихрений и разрежения за авто. В сочетании с повышенным давлением возникающее разрежение еще больше увеличивает сопротивление.
Поскольку повлиять на плотность воздуха невозможно, то конструкторам остается только вносить коррективы в две другие расчетные составляющие – площадь авто и коэффициент аэродинамического сопротивления.
Но уменьшить проекцию авто не представляется особо возможным без ущерба для полезных пространств кузова (просто невозможно сделать авто меньше, чем он есть), поэтому остается только изменение коэффициента Сх.
Этот коэффициент устанавливается экспериментальным путем (в аэродинамической трубе) и характеризует он соотношение лобового сопротивления к скоростному напору и площади поперечного сечения кузова. Величина его безразмерная.
Наименьший коэффициент аэродинамического сопротивления имеет каплевидное тело. При движении в воздушной массе такое тело плавно перед собой разводит поток, не создавая области повышенного давления, а имеющийся «хвост» позволяет за собой сомкнуть поток без обрывов и завихрений, то есть разрежение тоже отсутствует. Получается, что воздух просто обтекает тело, создавая минимальное сопротивление.
Для такого тела коэффициент Сх составляет всего 0,05.
Конструкторам, работая с аэродинамикой автомобиля добиться, таких показателей пока не удается. И все потому, что при движении сопротивление создается несколькими факторами:
- Формой кузова;
- Трением потока о поверхности при обтекании;
- Попаданием потока в подкапотное пространство и салон.
Поэтому для современных авто коэффициент аэродинамического сопротивления считается отличным, если его значение ниже 0,3. К примеру, у Peugeot 308 коэффициент составляет 0,29, у Audi A2 он равен 0,25, а у Toyota Prius – 0,26. Но стоит отметить, что это расчетные показатели в идеальных условиях. На практике же во время движения на авто воздействуют множество разнообразных факторов, которые негативным образом сказываются на сопротивлении кузова.
Примечательно, что на коэффициент оказывает наибольшее влияние не передок авто, а его задняя часть. И виной этому становится создание разрежения и завихрений в результате отрыва потока от кузова.
Поэтому конструкторы по большей части занимаются приданием необходимой формы именно задней части.
Коэффициент сопротивления Volkswagen XL1 составляет всего 0,19
Снизить коэффициент Сх позволяет также уменьшение количества выступающих частей, причем везде на авто (бока, крыша, днище, передок), а тем элементам, которые не удается убрать с поверхности придается максимально возможная обтекаемая форма.
Citroën GS
Вот еще один автомобиль, который при первом взгляде также не внушает доверия в аэродинамическое чудо, – это Citroën GS. На его премьере в 1970 году производитель объявил, что машина имеет коэффициент сопротивления воздуха всего 0,31 cd.
Семейный седан имел много места в комфортном салоне и оснащался гидропневматической подвеской. Было выпущено более 2,5 миллиона автомобилей. Выпуск продолжался до 1986 года.
Подъемная и прижимная сила
В результате неравномерного обтекания потоком воздуха автомобиля с разных сторон возникает разница в скорости его движения.
Действующие подъемная и прижимная силы
Автомобиль движется и рассекает поток воздуха, при этом часть этого потока уходит под авто и проходит под днищем, то есть движется практически по прямой. А вот верхней части потока приходится повторять форму кузова, и ей приходится проходить большее расстояние. Из-за этого возникает разница в скорости воздуха – верхняя часть движется быстрее нижней, проходящей под авто. А поскольку увеличение скорости сопровождается снижением давления, то под днищем образуется зона повышенного давления, которая приподнимает машину.
Проблем добавляет и лобовое сопротивление. Область повышенного давления воздушной массы перед машиной прижимает передок к дороге, в то время как разрежение и завихрения позади наоборот – способствуют приподнятию кузова. Подъемная сила, как и лобовое сопротивление, возрастает при увеличении скорости движения.
Но эта сила может оказывать и положительное действие. При внесении корректив в конструкцию авто возможно преобразование подъемной силы в прижимную, которая будет обеспечивать лучшее сцепление с дорогой, устойчивость авто, его управляемость на высоких скоростях.
При этом для получения прижимной силы не требуется каких-либо отдельных решений. Все разработки, направленные на снижение коэффициента Сх также сказываются и на прижиме. К примеру, оптимизация формы задней части приводит к уменьшению завихрений и разрежения, из-за чего подъемная сила тоже снижается, а прижимная — повышается. Установка заднего спойлера действует таким же образом.
casper76-68 › Блог › Аэродинамика отечественных автомобилей — ИЖ 2126 и ДРУГИХ.
Конечно все эти цифры не актуальны в сравнении с современными авто как российского так и импортного производства, но все же хочется малую толику внести в защиту иж 2126, который многие индивиды охаивают по чем зря.Плюс ко всему идет гнусная неправда сплошь и рядом как в обзорах «ваз классика против иж ода», так и в тестах новоявленных тестоводов…Поехали.
Поговорим немного об аэродинамических качествах отечественных автомобилей, конкретно о ИЖ 2126, и в том числе о ВАЗ-ах ЗАЗ-ах и АЗЛК.
Информацию о коэффициенте лобового сопротивления отечественных автомобилей практически не найти в интернете.
Но мы исправим этот пробел. Касательно Cx Автомобиля ИЖ 2126 на момент написания статьи поисковики выдают только журнал авторевю (2001 год №15). Cx там равен АЖ (!) (далее цитата жирным курсивом): 0,462, что, например, на 10 процентов больше, чем коэффициент Сх вазовской «девятки». От такой цифры (извините за выражение) у меня «глаза на лоб полезли».
Для справки некоторые данные о аэродинамическом сопротивлении отечественных авто: Cx ВАЗ 21013 = 0,46 [1] Cx ГАЗ 24-10 = 0,41 [1] Cx АЗЛК 2140 = 0,41 [1] Cx ИЖ 412-028 = 0,41 [1] Cx ИЖ 21251 = 0,39 [1] Cx ВАЗ 2108 = 0,38 [1] [2] Cx ВАЗ 2109 = 0,38 [2] Cx ЗАЗ 1102 = 0,37 [1] [2] Cx АЗЛК 2141 = 0,35 [2] Cx ИЖ 2126 = 0,35 [1] Cx ИЖ 2126 = 0,325 (со спойлером) [1]
[1] Данные из печатного источника (книга) «Твой друг автомобиль» М. Из-во ДОСААФ СССР 1988 год [2] Данные из журнала «За рулём» 1988 год №01 стр.8-9. (Данные совпадают с другими источниками 80-х, начала 90-х годов)
Читаем дальше тот-же номер авторевю: Основная причина плохой обтекаемости автомобиля — неровное дно, «уродующее» воздушный поток под машиной СТОП! Открываем книгу Ю.
Мацкерле «Современный экономичный автомобиль» М. Машиностроение 1987г. (Глава 4.4. Спойлеры) стр. 50: Уменьшенная (спойлером) скорость протекания воздуха по весьма неровному днищу кузова оказывает решающее влияние на снижение сопротивления воздуха. Не знаю как у вас но у меня создалось впечатление что я ездил на 2126-й из параллельного мира, сильно отличающийся от 2126-й которую «продували» где-то в другом параллельном мире авторевю… и 26-я из моего мира значительно лучше =)
Но это ещё не всё про спойлеры (терпение скоро закончим =) Там-же в авторевю: Велика и подъемная сила… Это грозит склонностью к заносам на высокой скорости. Ю. Мацкерле «Современный экономичный автомобиль» М. Машиностроение 1987г. (Глава 4.4. Спойлеры) стр. 50: В первую очередь — это (применение спойлера) увеличение прижатия передней оси к поверхности дороги с целью обеспечения хорошей управляемости автомобиля. Каждому опытному водителю известно — занос на заднеприводных автомобилях образуется (при движении) когда задние колёса продолжают толкать автомобиль, а передние потеряли контакт с дорогой.
Спойлер на бампере Иж 2126 для того изначально и придумали…
Кто виноват, и где ошибка?
Действительно «интересное кино»: по мнению журнала авторевю аэродинамика ИЖ 2126 хуже чем у Москвича Алеко, хуже чем у Таврии, и хуже чем у угловатых «восьмёрки» и «девятки». Мало того, они считают что обтекаемость ИЖа хуже чем у таких «динозавров» как М 412, 24-я Волга и даже (!) чем у «копейки», у которой передняя часть (фары, радиатор) стоит вертикально поперёк встречного ветра, а лобовое стекло даже при беглом взгляде имеет меньший угол наклона чем на ИЖ-е, а о отсутствии плавных (аэродинамически «зализанных») переходов на кузове «копейки» и следов нет! Однако, для журнала авторевю нет других источников кроме «себя самого». Они даже не упоминают что их результат О-ГО-ГО как противоречит другим источникам. Впрочем они не упоминают даже что они не единственные кто измерял Cx 2126-го ИЖа.а.
Что ещё влияет на аэродинамику?
Конечно, конструкторы стараются по максимуму снизить сопротивление авто при движении и повысить прижимную силу.
Но особенности эксплуатации авто и свой взгляд автовладельцев на внешние особенности машины вносят свои коррективы, причем в некоторых случаях – значительны.
Аэродинамическое сопротивление разных автомобилей в зависимости от скорости
К примеру, установка багажника на крышу, даже с аэродинамической формой увеличивает поперечную проекцию авто и сильно влияет на обтекаемость, это сразу сказывается на потреблении топлива.
Также расход повышается от езды с открытыми окнами и люком, использование защитных и декоративных обвесов, перевозка негабаритных грузов, выступающих за авто, нарушение положения конструктивных элементов, расположенных под днищем, повышение клиренса.
Но автовладелец также может и внести коррективы, которые положительно повлияют на аэродинамику автомобиля. К ним относится использование аэродинамических обвесов, установка спойлера, уменьшение клиренса.
Tatra 87
Представленная в 1936 году, Tatra 87 сегодня является иконой дизайна.
Благодаря хорошо спроектированной задней части машины значение аэродинамического сопротивления составляет 0,36. По традиции тех лет чешский автопроизводитель установил двигатель в заднюю часть машины.
Высокая скорость и низкое потребление топлива были сильной стороной Татры. Для того времени это был идеальный автомобиль для шоссе. К 1950 году было произведено 3000 автомобилей.
Сопротивление воздуха
На расход топлива, в особенности при больших скоростях движения, значительное влияние оказывает сопротивление воздуха (аэродинамическое сопротивление), сила аэродинамического сопротивления пропорциональна квадрату скорости и рассчитывается по формуле
где S – площадь фронтальной проекции автомобиля, м 2 ; v – скорость движения автомобиля относительно воздуха, м/с; ρ – плотность воздуха, кг/м 3 ; cх – коэффициент аэродинамического сопротивления.
Аэродинамическое сопротивление не зависит от массы автомобиля [2]. Площадь фронтальной проекции автомобиля определяется формой кузова и требованиям по обеспечению комфортного расположения водителя и пассажиров на сиденьях.
Например, автомобиль большого класса может быть ниже, чем малого, так как сиденья у него зачастую располагаются ниже. У автомобиля малого класса из-за его небольшой массы и длины сиденья расположены выше над полом, и поэтому расстояние между передними и задними сиденьями меньше. Более прямое расположение водителя и пассажиров в автомобиле малого класса требует его большей высоты, но меньшей длины. Площади фронтальных проекций обоих автомобилей при этом почти одинаковы, но низкий и длинный кузов автомобиля большого класса аэродинамически более выгоден.
Мощность двигателя, необходимая для преодоления аэродинамического сопротивления, пропорциональна, следовательно, кубу скорости:
где v — относительная скорость движения автомобиля, км/ч.
Коэффициент аэродинамического сопротивления, как видно из таблицы, представленной ниже, изменяется в широком диапазоне в зависимости от формы кузова автомобиля.
Аэродинамическое сопротивление различных автомобилей
| Кузов автомобиля | Коэффициент сопротивления воздуха cx | Мощность, необходимая для преодоления аэродинамического сопротивления (кВт), при площади фронтальной проекции 2 м 2 и скорости | ||
| 40 км/ч | 80 км/ч | 120 км/ч | ||
| Открытый четырёхместный | 0,7 – 0,9 | 1,18 – 1,47 | 9,6 – 11,8 | 31,0 – 40,5 |
| Закрытый, с наличием углов и граней | 0,6 – 0,7 | 0,96 – 1,18 | 8,0 – 9,6 | 26,4 – 30,8 |
| Закрытый, с закруглением углов и граней | 0,5 – 0,6 | 0,80 – 0,96 | 6,6 – 8,0 | 22,0 – 26,4 |
| Закрытый понтонообразный | 0,4 – 0,5 | 0,66 – 0,80 | 5,2 – 6,6 | 17,6 – 22,0 |
| Закрытый, хорошо обтекаемый | 0,3 – 0,4 | 0,52 – 0,66 | 3,7 – 5,2 | 13,2 – 17,6 |
| Закрытый, аэродинамически совершенный | 0,20 – 0,25 | 0,33 – 0,44 | 2,6 – 3,3 | 9,8 – 11,0 |
| Грузовой автомобиль | 0,8 – 1,5 | – | – | – |
| Автобус | 0,6 – 0,7 | – | – | – |
| Автобус с хорошо обтекаемым кузовом | 0,3 – 0,4 | – | – | – |
| Мотоцикл | 0,6 – 0,7 | – | – | – |
Коэффициент аэродинамического сопротивления устанавливается продувкой автомобиля или его макета в аэродинамической трубе или приближенно в ходе эксплуатационных испытаний.
При испытаниях в аэродинамической трубе на макетах получаются менее точные значения, чем при тех же испытаниях на реальных автомобилях. Это вызвано тем, что на изменение сопротивления воздуха оказывают влияние неточности изготовления некоторых узлов и деталей автомобиля: ручек дверей, днища кузова, бамперов, зеркал заднего вида и т. д. Кроме того, значительное влияние на величину сх оказывает воздух, проходящий в кузов для охлаждения и вентиляции.
При больших скоростях движения автомобиля аэродинамическое сопротивление является преобладающим.
На рисунке ниже показано изменение мощностей, необходимых для преодоления сопротивления качению Nf и аэродинамического сопротивления Nv в зависимости от скорости v для автомобиля среднего класса. При скорости 60 км/ч мощности, необходимые для преодоления сопротивления качению и сопротивления воздуха, равны, что характерно для данного вида автомобилей. По сумме потребляемых мощностей можно убедиться в важности сопротивления воздуха.
При скорости 80 км/ч мощность, затрачиваемая на его преодоление, в 4 раза больше, чем при скорости 40 км/ч, а при скорости выше, чем 120 км/ч, общая мощность, необходимая для движения, растет почти пропорционально кубу скорости автомобиля.
| Мощность, затрачиваемая на преодоление сопротивлений движению |
| Масса автомобиля 1350 кг, площадь фронтальной проекции S автомобиля 2 м 2 ; коэффициент сопротивления качению f равен 0,015; коэффициент аэродинамического сопротивления сх равен 0,456. |
При определении мощности двигателя, необходимой для достижения максимальной скорости, большей той, которую обеспечивает номинальная мощность установленного на автомобиле двигателя, можно использовать без значительной ошибки следующее соотношение:
где N2 – требуемая мощность, кВт; N1 – достигнутая максимальная мощность, кВт; v2 – требуемая скорость, км/ч; v1 – достигнутая максимальная скорость, км/ч.
Через точку X – максимальная мощность N1 при максимальной скорости v1 – проведена кривая зависимости мощности от куба скорости.
Разница между этой кривой и линией мощности, требуемой для движения при максимальной скорости, незначительна.
Citroën DS
Впервые представленный на Парижском автосалоне в 1955 году, Citroën DS выглядел для многих посетителей как космический корабль пришельцев, приземлившийся на Землю.
Чтобы подтвердить уникальность автомобиля, в дополнение к инновационной технологии (машина имела гидропневматическую подвеску!) дизайнеры создали модели футуристический аэродинамический дизайн, коэффициент сопротивления воздуха которого составлял 0,37. Это выдающийся результат по сравнению с конкурентами того времени.
Зачем это нужно
Для чего нужна аэродинамика автомобилю, знают все. Чем обтекаемее его кузов, тем меньше сопротивление движению и расход топлива. Такой автомобиль не только сбережет ваши деньги, но и в окружающую среду выбросит меньше всякой дряни. Ответ простой, но далеко не полный. Специалисты по аэродинамике, доводя кузов новой модели, еще и:
- рассчитывают распределение по осям подъемной силы, что очень важно с учетом немалых скоростей современных автомобилей,
- обеспечивают доступ воздуха для охлаждения двигателя и тормозных механизмов,
- продумывают места забора и выхода воздуха для системы вентиляции салона,
- стремятся понизить уровень шумов в салоне,
- оптимизируют форму деталей кузова для уменьшения загрязнения стекол, зеркал и светотехники.

Причем решение одной задачи зачастую противоречит выполнению другой. Например, снижение коэффициента лобового сопротивления улучшает обтекаемость, но одновременно ухудшает устойчивость автомобиля к порывам бокового ветра. Поэтому специалисты должны искать разумный компромисс.
ТОП-10 лучших автомобилей по аэродинамике
Статья про лучшие автомобили по аэродинамическим показателям: топ-10 моделей, их некоторые технические особенности. В конце статьи — видео про худшие машины по аэродинамике.
Чем меньше уровень аэродинамического сопротивления автомобиля, чем выше его предельная скорость, меньше расход топлива и стабильнее поведение на дороге. В сегодняшней подборке — автомобили, обладающие наилучшими аэродинамическими характеристиками.
Снижение лобового сопротивления
От чего зависит сила лобового сопротивления? Решающее влияние на нее оказывают два параметра – коэффициент аэродинамического сопротивления Сх и площадь поперечного сечения автомобиля (мидель).
Уменьшить мидель можно, сделав кузов ниже и уже, но вряд ли на такой автомобиль найдется много покупателей. Поэтому основным направлением улучшения аэродинамики автомобиля является оптимизация обтекания кузова, другими словами – уменьшение Сх. Коэффициент аэродинамического сопротивления Сх – это безразмерная величина, которая определяется экспериментальным путем. Для современных автомобилей она лежит в пределах 0,26-0,38. В зарубежных источниках коэффициент аэродинамического сопротивления иногда обозначают Cd (drag coefficient – коэффициент сопротивления). Идеальной обтекаемостью обладает каплевидное тело, Сх которого равен 0,04. При движении оно плавно рассекает воздушные потоки, которые затем беспрепятственно, без разрывов, смыкаются в его «хвосте».
Иначе ведут себя воздушные массы при движении автомобиля. Здесь сопротивление воздуха складывается из трех составляющих:
- внутреннего сопротивления при прохождении воздуха через подкапотное пространство и салон,
- сопротивления трения воздушных потоков о внешние поверхности кузова и
- сопротивления формы.

Третья составляющая оказывает наибольшее влияние на аэродинамику автомобиля. Двигаясь, автомобиль сжимает находящиеся перед ним воздушные массы, создавая область повышенного давления. Потоки воздуха обтекают кузов, а там, где он заканчивается, происходит отрыв воздушного потока, создаются завихрения и область пониженного давления. Таким образом, область высокого давления спереди мешает автомобилю двигаться вперед, а область пониженного давления сзади «засасывает» его назад. Сила завихрений и величина области пониженного давления определяется формой задней части кузова.
Передняя часть и боковые поверхности автомобиля особых хлопот конструкторам в плане аэродинамики не доставляют. Здесь главное – избегать резких переходов и выступов, предотвращая тем самым отрыв воздушного потока от поверхности кузова.
А вот с задней частью кузова все гораздо сложнее. Как нетрудно догадаться, наименее аэродинамичными являются универсалы – их форма меньше всего напоминает идеальную «каплю».
За их обширным «задком» образуется внушительная зона разряжения, которая не только снижает Сх, но и «засасывает» пыль и грязь, оседающую на заднем стекле. Немного уменьшить ее вредное воздействие можно с помощью установки дефлектора на верху пятой двери. Он направляет часть воздушного потока вниз, снижая разряжение и уменьшая загрязнение.
Не все просто и с хэтчбеками, хотя, на первый взгляд, их форма кажется наиболее обтекаемой. Впечатление обманчиво – яркий пример непредсказуемости аэродинамики. Сх хэтчбеков зависит от угла наклона задней части. При большом угле наклона (а таких моделей большинство) процесс обтекания практически не отличается от универсалов – воздушный поток отрывается от верхней кромки крыши и создает значительную зону разряжения.
С уменьшением угла наклона до 30-35 градусов точка отрыва потока перемещается на нижнюю кромку задней части. Казалось бы, зона разряжения и, соответственно, Сх должны уменьшиться. Но, как это на первый взгляд ни парадоксально, происходит все наоборот.
Дело в том, что в этом случае воздушные потоки с боков кузова, попадая на наклонную поверхность, образуют кромочные вихри, которые, закручиваясь по спирали, создают за автомобилем еще большую зону разряжения. Борются с этим явлением с помощью спойлера, устанавливаемого на кромке крыши. При этом точка отрыва потока перемещается с нижней кромки задней части на верхнюю, что предотвращает образование кромочных вихрей и несколько улучшает общую аэродинамику.
А вот если уменьшить наклон «задка» до 20-23 градусов, воздушный поток с крыши почти идеально обтекает автомобиль, отрываясь от нижней кромки. При этом кромочные вихри уже не образуются, и зона разряжения получается минимальной. Но такие автомобили теряют в практичности и поэтому среди серийных моделей их совсем немного.
Наилучшие показатели обтекаемости демонстрируют автомобили со ступенчатой формой задней части – седаны и купе. Объяснение простое – сорвавшийся с крыши поток воздуха тут же попадает на крышку багажника, где нормализуется и затем окончательно срывается с его кромки.
Боковые потоки тоже попадают на багажник, который не дает возникать вредным вихрям за автомобилем. Поэтому чем выше и длиннее крышка багажника, тем лучше аэродинамические показатели. На больших седанах и купе иногда даже удается достичь безотрывного обтекания кузова. Небольшое сужение задней части также помогает снизить Сх. Кромку багажника делают острой или в виде небольшого выступа – это обеспечивает отрыв воздушного потока без завихрений. В результате область разряжения за автомобилем получается небольшой.
Машины с лучшими аэродинамическими характеристиками
Mercedes-Benz CLA BlueEFFICIENCY
В 2013-м компания Mercedes представила специальную версию седана CLA BlueEFFICIENCY, при разработке которой огромное внимание было уделено аэродинамике.
Так, автомобиль получил специальные аэродинамические выштамповки, особый дизайн передних стоек и внешних зеркал, а также особый дизайн легкосплавных дисков. В результате величина сопротивления встречным потокам воздуха составила всего 0,22 Сх.
Tesla Model 3
Коэффициент Сх для электрокаров – один из наиболее значимых показателей, ведь чем он ниже, тем меньше автомобиль расходует электроэнергии и тем большее расстояние способен проехать.
В стандартном исполнении электрокар способен преодолеть 215 миль (346 км), при этом с нуля до сотни машина разгоняется за каких-то 6 сек.
Volkswagen XL1
В 2013 году Volkswagen показал модель XL1, которая, несмотря на свою футуристическую и, откровенно говоря, спорную внешность все же была запущена в серийное производство.
Всего было выпущено 250 экземпляров модели. Оправданием столь необычного дизайна стал низкий коэффициент аэродинамического сопротивления равный 0,19 Сх, что является самым лучшим результатом среди серийных автомобилей.
Daihatsu UFE-III Concept
В 2005 году руководство компании Daihatsu в рамках Токийской автовыставки продемонстрировало концептуальный автомобиль компакт-класса, получивший название UFE-III.
Под капотом авто располагался экономичный гибридный силовой агрегат, представленный 0,66-литровым бензиновиком и небольшим электродвигателем.
Модель могла похвастаться небольшим расходом топлива, не превышающим 1,6 л/100 км, а также отменной аэродинамикой – коэффициент лобового сопротивления равнялся всего 0,168 Сх.
General Motors Precept Concept
Precept Concept был представлен в 2002 году. Машина обладала необычной внешностью, выполненной в стилистике культового Citroen DS, а также скоромным аппетитом, не превышающим 3л/100 км.
При этом Precept мог похвастаться наличием 5-местного салона, а также коэффициентом аэродинамического сопротивления в 0,163 Сх.
К сожалению, производитель посчитал машину чрезмерно дорогой и сложной в конструировании, из-за чего было принято решение не пускать её в серийное производство.
Volkswagen 1 Liter Car Соncept
В 2002 году немецкий автоконцерн VW представил свою новую разработку – концептуальную модель 1 Liter Car Concept.
При создании авто перед производителями стояла задача создания максимально экономичного авто, и им это удалось. Средний расход топлива авто составил всего 0,99 л/100 км. Добиться такого показателя получилось за счёт небольшой массы (290 кг) и минимального аэродинамического сопротивления, составляющего всего 0,159 Сх.
JCB Dieselmax
В 2006 году дизельный JCB Dieselmax установил рекорд скорости, разогнавшись сначала до впечатляющих 529 км/ч, а потом до 563,42 км/час. Таким образом, машина смогла побить предыдущий рекорд в 380 км/ч, который был установлен в далёком 1973 году.
Заезды проводились на соляном озере Бонневиль, расположенном на территории штата Юта (США).
Автомобиль мог похвастать обтекаемым кузовом, имеющим коэффициент аэродинамического сопротивления в 0,147 Сх, а также парой дизельных двигателей, устанавливаемых на экскаваторах.
В настоящее время автомобиль хранится в музее компании JCB.
Fiat Turbina
В 1954 году итальянский автопроизводитель Fiat представил модель Turbina, ставшей первым европейским авто с газотурбинным двигателем.
Максимальная отдача силовой установки достигала 300 л. с., а максимальная скорость достигала отметки в 250 км/ч.
Однако самой главной особенностью модели был её аэродинамически высокоэффективный кузов, величина аэродинамического сопротивления которого составляла всего 0,14 Сх.
Несмотря на наличие первоклассной аэродинамики, машина была признана бесперспективной и отправлена на хранение в Туринский автомузей, где она находится по сегодняшний день.
Ford Probe V Concept
В 1983 году компания Ford начала разработку концепт-кара Probe V Concept, официальный дебют которого состоялся в 1985 году.
Машина обладала футуристической внешностью со сдвижными боковыми дверьми. Кроме того инженеры закрыли колеса специальными щитками, а стекла вклеили в оконные проёмы заподлицо с поверхностью кузова.
Но несмотря на все усилия разработчика, автомобиль так и не пошёл в серийное производство, оставшись необычным и стильным концептом.
Goldenrod Land Speed Race Car
Goldenrod Land Speed Race Car был сконструирован братьями Саммерсами в далёком 1965 году, при этом автомобиль по сегодняшний день носит звание самого аэродинамичного в мире.
Так, показатель лобового сопротивления «автомобильной торпеды» составляет всего 0,117 Сх. В движение машина приводилась посредством 4-х семилитровых 8-цилиндровых бензиновых моторов, расположенных продольно друг за другом и суммарно генерирующих мощность 2400 л. с.
Заключение
Борьба за лучшую аэродинамику продолжается, а значит, уже совсем скоро мы можем увидеть концепт или серийную версию авто, аэродинамические показатели которого смогут превзойти показатели Goldenrod Land Speed Race Car.
Видео про худшие машины по аэродинамике:
Источник
Прижимная сила
При движении автомобиля поток воздуха под его днищем идет по прямой, а верхняя часть потока огибает кузов, то есть, проходит больший путь. Поэтому скорость верхнего потока выше, чем нижнего. А согласно законам физики, чем выше скорость воздуха, тем ниже давление. Следовательно, под днищем создается область повышенного давления, а сверху – пониженного. Таким образом создается подъемная сила.
И хотя ее величина невелика, неприятность состоит в том, что она неравномерно распределяется по осям. Если переднюю ось подгружает поток, давящий на капот и лобовое стекло, то заднюю дополнительно разгружает зона разряжения, образующаяся за автомобилем. Поэтому с ростом скорости снижается устойчивость и автомобиль становится склонен к заносу.
Каких-либо специальных мер для борьбы с этим явлением конструкторам обычных серийных автомобилей выдумывать не приходится, так как то, что делается для улучшения обтекаемости, одновременно увеличивает прижимную силу. Например, оптимизация задней части уменьшает зону разряжения за автомобилем, а значит и снижает подъемную силу. Выравнивание днища не только уменьшает сопротивление движению воздуха, но и повышает скорость потока и, следовательно, снижает давление под автомобилем. А это, в свою очередь, приводит к уменьшению подъемной силы. Точно так же две задачи выполняет и задний спойлер. Он не только уменьшает вихреобразование, улучшая Сх, но и одновременно прижимает автомобиль к дороге за счет отталкивающегося от него потока воздуха.
Иногда задний спойлер предназначают исключительно для увеличения прижимной силы. В этом случае он имеет большие размеры и наклон или делается выдвижным, вступая в работу только на высоких скоростях.
Для спортивных и гоночных моделей описанные меры будут, естественно, малоэффективны. Чтобы удержать их на дороге, нужно создать большую прижимную силу. Для этого применяются большой передний спойлер, обвесы порогов и антикрылья. А вот установленные на серийных автомобилях, эти элементы будут играть только лишь декоративную роль, теша самолюбие владельца. Никакой практической выгоды они не дадут, а наоборот, увеличат сопротивление движению. Многие автолюбители, кстати, путают спойлер с антикрылом, хотя различить их довольно просто. Спойлер всегда прижат к кузову, составляя с ним единое целое. Антикрыло же устанавливается на некотором расстоянии от кузова.
Mercedes-Benz А-Класс седан
К концу нынешнего десятилетия (на данный момент) самым аэродинамическим автомобилем на рынке является седан Mercedes A-класса (модельный ряд 2021 года) с исключительной аэродинамикой (коэффициент 0,22 cd).
Это стало возможным благодаря комплексной герметизации кузова автомобиля (включая герметизацию фар), включая полную герметизацию днища автомобиля. В том числе полностью изолирован моторный отсек, детали задней подвески и многое другое. Спойлеры колес сзади и спереди были специально оптимизированы, чтобы колеса могли вращаться с минимальными потерями.
Как и зачем улучшают аэродинамические характеристики автомобиля
В процессе проектирования и создания конструкторами очень тщательно прорабатывается аэродинамика автомобиля, поскольку она оказывает значительное влияние на технические показатели модели.
При движении автомобиля большая часть мощности силовой установки уходит на преодоление сопротивления, создаваемого воздухом. И правильно созданная аэродинамика автомобиля позволяет уменьшить это сопротивление, а значит на борьбу с противодействием находящего воздушного потока потребуется затратить меньше мощности, и соответственно – топлива.
Измерение аэродинамики автомобиля проводится для изучения сил, создаваемых воздушным потоком и воздействующих на транспортное средство. И таких сил несколько – подъемные и боковые, а также лобовое сопротивление.
Лобовое сопротивление и коэффициент Сх
По большей части все работы с кузовом авто направлены на преодоление лобового сопротивления, поскольку именно эта сила самая значительная.
Движение потоков воздуха
За основу при расчетах берется сила сопротивления воздуха. Для вычисления результата используются такие данные как плотность воздуха, площадь поперечной проекции авто, коэффициент аэродинамического сопротивления (Сх) — это важнейший показатель в аэродинамике автомобиля. При этом на силу сопротивления в значительной мере влияет также скорость движения. Так, увеличение скорости вдвое будет сопровождаться повышением сопротивлением в 4 раза. Скорость один из мощных факторов увеличения расхода.
Например, для хорошо обтекаемого авто с площадью проекции 2 м2 и коэффициентом 0,3 при движении на скорости 60 км/ч для преодоления сопротивления воздуха необходимо 2,4 л.
с., а при скорости 120 км/ч уже 19,1 л.с. Разница расхода топлива при таких условиях достигает 30% на 100 км.
Если вам, в данный момент, требуется максимальная экономия топлива, необходимо придерживаться постоянной скорости около 60 км/ч. В этом режиме движения расход будет минимальным даже у авто с большим Cx.
Рассмотрим все по-простому. У воздуха есть своя плотность, причем немалая. При движении автомобилю приходится проходить через имеющиеся воздушные массы, при этом создается поток, который обтекает кузов. И чем легче авто будет «резать» воздушную массу, тем меньше он затратит на это энергии.
Но не все так просто. Во время движения перед авто создается область увеличенного давления (машина сжимает воздушную массу), то есть спереди образуется такой себе невидимый барьер, осложняющий «разрезание» воздушной массы.
Также после обтекания кузова происходит отрыв воздушного потока от поверхности, что становиться причиной появления завихрений и разрежения за авто.
В сочетании с повышенным давлением возникающее разрежение еще больше увеличивает сопротивление.
Поскольку повлиять на плотность воздуха невозможно, то конструкторам остается только вносить коррективы в две другие расчетные составляющие – площадь авто и коэффициент аэродинамического сопротивления.
Но уменьшить проекцию авто не представляется особо возможным без ущерба для полезных пространств кузова (просто невозможно сделать авто меньше, чем он есть), поэтому остается только изменение коэффициента Сх.
Этот коэффициент устанавливается экспериментальным путем (в аэродинамической трубе) и характеризует он соотношение лобового сопротивления к скоростному напору и площади поперечного сечения кузова. Величина его безразмерная.
Аэродинамическая труба
Наименьший коэффициент аэродинамического сопротивления имеет каплевидное тело. При движении в воздушной массе такое тело плавно перед собой разводит поток, не создавая области повышенного давления, а имеющийся «хвост» позволяет за собой сомкнуть поток без обрывов и завихрений, то есть разрежение тоже отсутствует.
Получается, что воздух просто обтекает тело, создавая минимальное сопротивление. Для такого тела коэффициент Сх составляет всего 0,05.
Конструкторам, работая с аэродинамикой автомобиля добиться, таких показателей пока не удается. И все потому, что при движении сопротивление создается несколькими факторами:
- Формой кузова;
- Трением потока о поверхности при обтекании;
- Попаданием потока в подкапотное пространство и салон.
Поэтому для современных авто коэффициент аэродинамического сопротивления считается отличным, если его значение ниже 0,3. К примеру, у Peugeot 308 коэффициент составляет 0,29, у Audi A2 он равен 0,25, а у Toyota Prius – 0,26. Но стоит отметить, что это расчетные показатели в идеальных условиях. На практике же во время движения на авто воздействуют множество разнообразных факторов, которые негативным образом сказываются на сопротивлении кузова.
Примечательно, что на коэффициент оказывает наибольшее влияние не передок авто, а его задняя часть.
И виной этому становится создание разрежения и завихрений в результате отрыва потока от кузова. Поэтому конструкторы по большей части занимаются приданием необходимой формы именно задней части.
Коэффициент сопротивления Volkswagen XL1 составляет всего 0,19
Снизить коэффициент Сх позволяет также уменьшение количества выступающих частей, причем везде на авто (бока, крыша, днище, передок), а тем элементам, которые не удается убрать с поверхности придается максимально возможная обтекаемая форма.
Немного теории
Коэффициент аэродинамического сопротивления автомобиля указывается в величине Cx, обычно она меньше 1. Чем он будет меньше, тем меньше мощностей он будет затрачивать для движения. Так показатель Cx у AUDI A8 — 0.37, Lexus LS 460 — 0.26. Весьма странным может показаться тот факт, что у спорткаров этот показатель значительно выше (Porsche 911 Turbo 997 — 0.31, Bugatti Veyron — 0.42). На самом же деле все довольно просто. Мощные двигатели требуют охлаждения, в том числе и воздушными потоками.
Добиться этого можно увеличив площадь радиатора, а значит и поперечное сечение машины.
Подъемная и прижимная сила
В результате неравномерного обтекания потоком воздуха автомобиля с разных сторон возникает разница в скорости его движения.
Действующие подъемная и прижимная силы
Автомобиль движется и рассекает поток воздуха, при этом часть этого потока уходит под авто и проходит под днищем, то есть движется практически по прямой. А вот верхней части потока приходится повторять форму кузова, и ей приходится проходить большее расстояние. Из-за этого возникает разница в скорости воздуха – верхняя часть движется быстрее нижней, проходящей под авто. А поскольку увеличение скорости сопровождается снижением давления, то под днищем образуется зона повышенного давления, которая приподнимает машину.
Проблем добавляет и лобовое сопротивление. Область повышенного давления воздушной массы перед машиной прижимает передок к дороге, в то время как разрежение и завихрения позади наоборот – способствуют приподнятию кузова.
Подъемная сила, как и лобовое сопротивление, возрастает при увеличении скорости движения.
Негативным фактором от воздействия такой силы является ухудшение устойчивости авто при увеличении скорости и повышение вероятности ухода в занос.
Но эта сила может оказывать и положительное действие. При внесении корректив в конструкцию авто возможно преобразование подъемной силы в прижимную, которая будет обеспечивать лучшее сцепление с дорогой, устойчивость авто, его управляемость на высоких скоростях.
При этом для получения прижимной силы не требуется каких-либо отдельных решений. Все разработки, направленные на снижение коэффициента Сх также сказываются и на прижиме. К примеру, оптимизация формы задней части приводит к уменьшению завихрений и разрежения, из-за чего подъемная сила тоже снижается, а прижимная — повышается. Установка заднего спойлера действует таким же образом.
Уменьшение завихрений при установке спойлера
Боковые же силы при установлении аэродинамики автомобиля, особо в расчет не берутся, в силу того, что они не постоянны, а также значительного влияния на показатели авто не оказывают.
Но это все теория аэродинамики автомобиля. На практике все можно пояснить одним предложением — чем хуже аэродинамика, тем выше расход топлива.
Как меняют аэродинамику автомобиля?
Задача специалистов по аэродинамике состоит в уменьшении паразитных сил и моментов (Рх, Рz, Му, Мх и Мz). Добиться можно с помощью дополнительных аэродинамических элементов, что ведет к увеличению площади миделя и как следствие – к увеличению силы лобового сопротивления. Тупик? Нет, оказывается, грамотно сконструированные и тщательно продутые в аэродинамической трубе элементы позволяют уменьшить Сх! Что это за устройства? Обычно при слове обвес речь идет о бамперах, порогах, спойлерах и антикрыльях.
Антикрыло. Создано для борьбы с подъемной силой. Первостепенная задача – создать прижимную силу, чтобы колеса не теряли контакт с дорогой ни при каких условиях. Взгляните на болиды Ф1. Вот где антикрылья – усилия работы специалистов по аэродинамике! Но перебарщивать с размерами нельзя – резко растет аэродинамическое сопротивление, а значит – падает скорость, увеличивается расход топлива.
Практически на всех спортивных автомобилях рабочая часть крыла выполнена регулируемой для возможности изменения угла атаки и возможности настройки.
Вискомуфта (вязкостная муфта). Принцип работы и устройство
Спойлер (от spoil — портить). Аэродинамический элемент с одной рабочей поверхностью для изменения направления движения воздушного потока. Основная задача «правильного» спойлера – организация безотрывного и «плавного» обтекания воздушным потоком всей поверхности автомобиля, что повышает устойчивости при движении с высокими скоростями. Спойлер может бороться с подъемной силой, отсюда его сложные формы. Но эта деталь всегда примыкает к кузову автомобиля. По большому счету, бамперы и пороги это тоже большие спойлеры.
Спойлер и антикрыло – основные, но не единственные элементы, улучшающие аэродинамику. Если заглянуть под днище современного авто, то увидим большое количество специальных щитков. Их задача – уменьшить сопротивление, исключить завихрения и направление потока в нужном направлении.
Иногда проработка днища дает потрясающие результаты.
Диффузор. Дальше всех пошли спортсмены – они решили присосать автомобиль к трассе! Появились болиды с днищем, имитирующим «трубку Вентури» – создающие резкий рост скорости воздушного потока под машиной. В результате создавалась мощная прижимная сила. Плодами этого открытия норовит воспользоваться каждый автопроизводитель: диффузоры, обеспечивающие ускорение потока, появляются в задней части гражданских машин.
Проблема, что для максимально эффективной реализации т.н. «граунд-эффекта» нужны по возможности плоское днище и минимальный дорожный просвет. Если строители спортивных машин могут это позволить, то, к примеру, на Evolution диффузор служит скорее украшением, чем полноценным аэродинамическим элементом.
Турбокомпрессор: что это такое и как он работает
Что ещё влияет на аэродинамику?
Конечно, конструкторы стараются по максимуму снизить сопротивление авто при движении и повысить прижимную силу.
Но особенности эксплуатации авто и свой взгляд автовладельцев на внешние особенности машины вносят свои коррективы, причем в некоторых случаях – значительны.
Аэродинамическое сопротивление разных автомобилей в зависимости от скорости
К примеру, установка багажника на крышу, даже с аэродинамической формой увеличивает поперечную проекцию авто и сильно влияет на обтекаемость, это сразу сказывается на потреблении топлива.
Также расход повышается от езды с открытыми окнами и люком, использование защитных и декоративных обвесов, перевозка негабаритных грузов, выступающих за авто, нарушение положения конструктивных элементов, расположенных под днищем, повышение клиренса.
Но автовладелец также может и внести коррективы, которые положительно повлияют на аэродинамику автомобиля. К ним относится использование аэродинамических обвесов, установка спойлера, уменьшение клиренса.
ALFA 40-60 HP Aerodinamica Castagna
Первым в истории шоу-каром и первой попыткой применить принципы аэродинамики к автомобилям был аэродинамический автомобиль ALFA, выпущенный в 1914 году (в те годы марка еще не называлась Alfa Romeo).
Смотрите также: Необычные автомобили, которые приехали на техосмотр
Автомобиль был создан итальянской компанией Carrozzeria Castagna для графа Марио Рикотти. Кузов машины был выполнен в виде капли и опирался на классическую раму.
Благодаря алюминиевому кузову и отсутствию капота максимальная скорость этого концепта составляла 120 км/ч. Когда машина пошла в серийный выпуск, скорость уже составляла 139 км/ч. К сожалению, точное значение аэродинамического сопротивления воздуха этого автомобиля неизвестно.
Что такое сопротивление качению шины?
Давайте разберемся, что такое сопротивление качению шин. Когда вы нажимаете на педаль газа в вашем автомобиле, вы начинаете ускоряться. Но если посмотреть более детально на сам процесс ускорения автомобиля, то можно увидеть, что нажимая на педаль газа вы передаете энергию от сгорания топлива в моторе, или электрическую энергию (все зависит от того, какой тип двигателя вы используете) через другие системы прямиком на шины вашего автомобиля.
Это приводит к тому, что ваши покрышки начинают оборачиваться и набирать достаточный импульс, чтобы ваш автомобиль начал двигаться. Но для того, чтобы колесо начало двигаться, ваши покрышки должны преодолеть очень много факторов, которые препятствуют началу движения. И одним из этих факторов является сопротивление качению шины.
Если говорить техническим языком, то сопротивление качению шины – это минимальная энергия, которую ваше транспортное средство должно передать на колеса, чтобы поддерживать постоянную скорость на ровном дорожном полотне. Другими словами, это усилие, которое нужно для того, чтобы колесо постоянно двигалось.
Главным источником сопротивлению качения является процесс, который называется гистерезис. С технической точки зрения, гистерезис – это, по сути, потеря энергии, которая возникает при прохождении шины по поверхности дорожного полотна. Из-за того, что двигатель автомобиля должен постоянно компенсировать гистерезис, он должен вырабатывать дополнительную энергию, что приводит к увеличению расхода топлива.
Минимальное сопротивление качению шины
Скажем сразу, что избавиться сопротивления качению шины нельзя исходя из законов физики. Но раз его нельзя полностью устранить, его можно попробовать свести к минимуму. Как мы уже выяснили, сопротивление качения шин вызвано гистерезисом. Чтобы минимизировать его влияние можно спроектировать покрышку таким образом, чтобы ее протектор был как можно меньше. Минусом такого подхода будет очень малый срок службы покрышки.
Второй способ более технологичен, ведь он требует разработки качественно новых материалов для шин. Покрышки из таких материалов получат снижение сопротивления качению благодаря тому, что они будут устойчивыми к выработке тепла и их протектор будет иметь минимальный прогиб во время сцепления с дорожным покрытием. Современные шины с низким сопротивлением качению используют именно второй подход.
Коэффициент аэродинамического сопротивления во Flow simulation 2016
Вступление.
Добрый день, дорогие читатели.
В данном посте я хочу рассказать, как посредствам внутреннего анализа во Flow simulation выполнить внешний анализ детали или конструкции на определения коэффициента аэродинамического сопротивления и результирующей силы. Так же рассмотреть создание локальной сетки и задание целей ‘цель-выражение’ для упрощения и автоматизации расчетов. Приведу основные понятия по коэффициенту аэродинамического сопротивления. Все эти сведения помогут быстро и грамотно спроектировать бедующее изделия и в дальнейшем распечатать его для практического использования.
Матчасть.
Коэффициент аэродинамического сопротивления (далее КАС) определяется экспериментально при испытаниях в аэродинамической трубе или испытаниях при движении накатом. Определение КАС приходит с формулой 1
формула 1
КАС разных форм колеблется в широком диапазоне. Рисунок 1 показывает эти коэффициенты для ряда форм. В каждом случае предполагается, что воздух, набегающий на тело, не имеет боковой компоненты (то есть движется прямо вдоль продольной оси транспортного средства).
Рисунок 1.
В жизни в дополнение к составляющей ветра, вытекающей из скорости движения автомобиля, учитывают скрость находящего ветра на автомобиль. И того для определения скорости потока верно следующее утверждение V=Vавто+Vветра.
Если находящий ветер является попутным то скорость вычитается.
Коэффициент аэродинамического сопротивления нужен для определения аэродинамического сопротивления, но в данной статье будет рассматриваться только сам коэффициент.
Исходные данные.
Расчет выполнялся в Solidworks 2016, модуль Flow simulation (далее FS).
В качестве исходных данных были взяты следующие параметры: скорость вытекающая из скорости движения автомобиля V=40 м/с, температура окружающей среды плюс 20 градусов Цельсия, плотность воздуха 1,204 кг/м3. Геометрическая модель автомобиля представлена упрощенно (см. рисунок 2).
Рисунок 2.
Шаги задания начальных и граничных условий во Flow simulation.
Процесс добавления модуля FS и общий принцип формирования задания на расчет описан в этой статье, я же опишу характерные особенности для внешнего анализа посредствам внутреннего.
1.На первом шаге добавляем модель в рабочее пространство.
Рисунок 2.
2. Далее моделируем аэродинамическую камеру прямоугольного сечения. Главная особенность при моделирование это отсутствие торцов, иначе мы не сможем задать граничные условия. Модель автомобиля должна находится в центре. Ширина трубы должна соответствовать 1,5* ширины модели в обе стороны, длина трубы 1,5*длины модели, от задней части модели и 2*длины автомобиля от бампера, высота трубы 1,5*высоты машины от плоскости на которой стоит машина.
Рисунок 3.
3. Входим в модуль FS. Задаём граничные условия на первой грани входной поток.
Рисунок 4.
Выбираем тип: расход/скорость->скорость на входе. Задаём нашу скорость. Выбираем параллельную грань к передней части авто. Нажимаем галочку.
Рисунок 5.
Задаём граничное условие на выходе. Выбираем тип: давление, всё оставляем по умолчанию. Жмём галку.
Итак, граничные условия заданы переходим к заданию на расчёт.
4. Нажимаем на мастер проекта и следуем инструкции по рисункам ниже.
Рисунок 6.
Рисунок 7.
Рисунок 8.
Рисунок 9.
Рисунок 10.
Рисунок 11.
В пункте завершение оставляем всё без изменений. Нажимаем завершить.
5. На этом шаге займёмся управлением и созданием локальной сетки. Нажимаем на дереве элементов FS на пункт: сетка, правой кнопкой мыши и выбираем: добавить локальную сетку.
Рисунок 12.
Рисунок 13.
Здесь можно указать параметры и область локальной сетки, для сложных моделей так же задаётся угол кривизны и минимальный размер элемента.
Минимальный размер задаётся в графе ‘закрытие узкие щели’. Данная функция существенно сокращает время расчета и увеличивает точность полученных данных. В зависимости от того, насколько точно вы хотите получить результаты, выставляется параметр дробление сетки. Для внутреннего анализа вполне подходят стандартные настройки. Далее будет показана визуализация сетки на поверхности.
6.Перед тем как запустить расчет нужно задать цели расчета. Цели задаются в дереве FS цели. В начале задаём глобальные цели, выбираем силы по каждой компоненте.
Рисунок 14.
После нам нужно задать ‘цели-выражения’. Для этого щелкаем правой кнопкой мыши в дереве FS на цели и выбираем ‘цель выражение’. Для начала зададим уравнения для результирующей силы .
Рисунок 15.
Что бы компанента по силе использовалась в выражение нужно щёлкнуть на неё левой кнопкой мыши , ссылка на компоненту появится в формуле. Здесь вводим формулу 2. Нажимаем на галку.
Формула 2.
Создаём вторую ‘цель-выражение’, записываем туда формулу 1.![]()
Рисунок 16.
КАС расчтывается для лобового стекла. В данной модели лобовое стекло это наклонная грань, грань наклонена на 155 градусов, поэтому сила по X умножается на sin(155*(пи/180)). Нужно помнить, что расчет ведётся по системе си и соответственно площадь наклонной грани должна измеряться в метрах квадратных.
7. Теперь можно приступить к расчету, запускаем расчет.
Рисунок 17.
При запуске расчета программа предоставляет выбор на чем производить расчет, мы можем выбрать количество ядер участвующие в расчете и рабочие станции.
Рисунок 18.
Так как задача не сложная расчет проходит меньше чем за минуту, поэтому мы нажмём на паузу после его запуска.
Рисунок 19.
Теперь нажимаем на кнопку ‘вставить график’, выбираем наши цели выражения.
Рисунок 20.
На графике будут показаны значения для наших выражений для каждой итерации.
Для наблюдения происходящего процесса во время расчета можно использовать ‘предварительный просмотр’.
Рисунок 21.
Рисунок 22.
То что эпюра перевёрнута нет ни чего страшного, это зависит от ориентации модели.
Расчёт заканчивается когда все цели сошлись.
Рисунок 23.
Результаты должны загрузиться автоматически, если этого не произошло догрузите вручную: инструменты->FS->результаты->загрузить из файла
8. После расчета можно посмотреть сетку на модели.
Рисунок 24.
Рисунок 25.
Выбираем грани где хотим видеть сетку.
Рисунок 26.
Такая сетка нам подходим, одна ячейка не больше самого маленького элемента в модели.
9. Теперь мы можем визуализировать полученные результаты.
Рисунок 27.
Рисунок 28.
Рисунок 29.
Мы можем наглядно увидеть и оценить правильность результатов. Под правильностью я подразумеваю, что бы они соответствовали действительным физическим процессам.
10. На красивых картинках дело не заканчивается. Для нас важно знать числовые параметры и уметь их извлекать из расчета. Ниже рассмотрено как результаты импортировать в экс ель.
Рисунок 30.
Рисунок 31.
Рисунок 32.
Эти данные можно использовать для проведение исследования зависимости угла наклона лобового стекла и величины КАС. Все результаты расчета во FS можно импортировать в другие расчетные комплексы и использовать уже в качестве входных данных, расчетов на прочность и т.д.
На этом можно закончить статью, если будут пожелания рассмотреть какой либо еще анализ или по глубже капнуть в аэродинамику, обязательно исполню.
Коэффициент аэродинамического сопротивления автомобиля — Вики
Коэффицие́нт аэродинами́ческого сопротивле́ния — безразмерная величина, равная отношению силы лобового сопротивления автомобиля F{\displaystyle F} к произведению скоростного напора Q{\displaystyle Q} на площадь миделевого сечения автомобиля S{\displaystyle S}.
{2}}{2}}S.}
Коэффициент аэродинамического сопротивления сферы в зависимости от числа Рейнольдса. Приведены графики для гладкой и шероховатой сфер. Развитая турбулентность потока у гладкой сферы развивается при бо́льших скоростях потока.
Cx{\displaystyle C_{x}} зависит только от формы автомобиля и числа Рейнольдса, при равенстве всех критериев подобия, в данном случае существенно число Рейнольдса, одинаков для всех геометрически подобных тел, независимо от их конкретных размеров. Cx{\displaystyle C_{x}} в широком диапазоне чисел Рейнольдса (Re), от ~1000 до ~10 5 приблизительно постоянно. При малых Re Cx{\displaystyle C_{x}} увеличивается из-за перехода обтекающего потока в ламинарное течение, для автомобиля такое Re соответствует скорости нескольким десяткам сантиметрам в секунду. При Re>105 наступает полное развитие турбулентности как на лобовой, так и на тыльной сторонах обтекаемого тела и Cx{\displaystyle C_{x}} снижается.
Чем меньше Cx{\displaystyle C_{x}}, тем меньше лобовое сопротивление движению автомобиля и меньше расход топлива при прочих равных условиях. Cx{\displaystyle C_{x}} современных легковых серийно выпускаемых автомобилей лежит в пределах от 0,2 до 0,35. У грузовых автомобилей и внедорожников, из-за плохо обтекаемого воздухом массивного кузова — до 0,5 и более.
Некоторые производители указывают в спецификациях эффективную площадь сопротивления автомобиля Seff{\displaystyle S_{eff}}:
- Seff=Cx⋅S.{\displaystyle S_{eff}=C_{x}\cdot S.}
Эта величина равна площади тонкой плоской пластины, ориентированной перпендикулярно набегающему потоку и испытывающей равную силу сопротивления с автомобилем, движущемся с той же скоростью, так как Cx{\displaystyle C_{x}} тонкой пластины близок к 1. Эффективная площадь зависит не только от формы, но и от размеров автомобиля, точнее, от площади его миделева сечения. Эффективная площадь современных серийных автомобилей составляет от 0,5 м2 для легковых до 2 и более квадратных метров у внедорожников и грузовиков.
Коэффициент сопротивления определяется экспериментальным путём продувкой макетов автомобилей в аэродинамической трубе, либо расчётным путём с помощью компьютерного моделирования.
Содержание
- 1 Мощность двигателя, затрачиваемая на преодоление сопротивления воздуха
- 2 Примеры
- 2.1 Серийно выпускаемые автомобили
- 2.2 Несерийные и уникальные автомобили
- 3 См. также
- 4 Ссылки
Мощность двигателя, затрачиваемая на преодоление сопротивления воздуха
Мощность, затрачиваемая на перемещение тела с силой F{\displaystyle F} равна произведению этой силы на скорость v:{\displaystyle v:}
- Pa=F⋅v.{\displaystyle P_{a}=F\cdot v.}
Так как сила аэродинамического сопротивления пропорциональна квадрату скорости, то часть мощности двигателя, идущей на преодоление сопротивления воздуху пропорциональна кубу скорости, т. е увеличение скорости в два раза требует увеличения мощности на преодоление сопротивления в восемь раз:
- Pa=Cxρv32S=ρv32Seff.
{3}}{2}}S_{eff}.}
- Пример
У автомобиля в летний день (плотность воздуха ~1,2 кг/м3), с эффективной площадью 1 м2, движущегося со скоростью 10 м/с (36 км/час) двигатель затрачивает на преодоление сопротивления воздуха около 600 Вт, а при движении со скоростью 30 м/с (108 км/час) уже ~16 кВт (~22 л. с.).
Примеры
Некоторые примеры коэффициентов аэродинамического сопротивления современных автомобилей:
Серийно выпускаемые автомобили
Cx=0,29{\displaystyle C_{x}=0,29} — Peugeot 308, 2007
Cx=0,28{\displaystyle C_{x}=0,28} — Porsche 997, 2004
Cx=0,27{\displaystyle C_{x}=0,27} — Infiniti G35, 2002 (Cx=0,26{\displaystyle C_{x}=0,26} «aero package»)
Cx=0,27{\displaystyle C_{x}=0,27} — Toyota Camry Hybrid, 2007
Cx=0,26{\displaystyle C_{x}=0,26} — Mercedes-Benz W221 S-Class, 2006
Cx=0,26{\displaystyle C_{x}=0,26} — Lexus LS 430, 2001 (0,25 air suspension)
Cx=0,26{\displaystyle C_{x}=0,26} — Toyota Prius, 2004
Cx=0,25{\displaystyle C_{x}=0,25} — Audi A2 1.
2 TDI, 2001Cx=0,24{\displaystyle C_{x}=0,24} — Tesla Model S, 2013
Cx=0,40{\displaystyle C_{x}=0,40} — Toyota Land Cruiser, 1991
Несерийные и уникальные автомобили
Cx=0,2{\displaystyle C_{x}=0,2} — Loremo, 2007
Cx=0,195{\displaystyle C_{x}=0,195} — General Motors EV1, 1996
Cx=0,19{\displaystyle C_{x}=0,19} — Mercedes-Benz Bionic, 2005
Cx=0,18{\displaystyle C_{x}=0,18} — Acabion, 2006
Cx=0,18{\displaystyle C_{x}=0,18} — Мерседес Т80, 1939
Cx=0,25−0,19{\displaystyle C_{x}=0,25-0,19} — Mercedes-Benz Concept IAA, 2015
См. также
- Лобовое сопротивление
- Турбулентное течение
- Каммбэк
- Спойлер
- Аэродинамика автомобиля
Ссылки
- Унесенные ветром: Аэродинамика автомобилей, autotechnic.su, 30.10.2009
- Кадр дня: Сверхлегкий и сверхэкономичный автомобиль, Иван Карташев, 21 июня 2007 (Loremo AG: LS / GT)
Худшие автомобили по аэродинамике: ТОП-7
Автомобили с худшими показателями по аэродинамике: топ-7 моделей, их особенности и некоторые важные технические характеристики.
В конце — видео про аэродинамику автомобилей в Формуле 1.Автомобили с худшими показателями по аэродинамике: топ-7 моделей, их особенности и некоторые важные технические характеристики. В конце — видео про аэродинамику автомобилей в Формуле 1.
Содержание статьи:
- Рейтинг автомобилей с худшей аэродинамикой
- Видео про аэродинамику автомобилей в Формуле 1
Автопроизводители неустанно совершенствуют свои модели, делая их быстрее, мощнее, технологичнее. Одним из параметров, по которому оценивают автомобиль, является аэродинамика. Обтекаемость корпуса напрямую влияет не только на скорость, но и на расход топлива, маневренность и устойчивость.
Рассмотрим в нашем обзоре модели, имеющие худшую аэродинамику.
Автомобили с действительно слабой аэродинамикой можно найти у каждого производителя, хотя официально открывается эта информация весьма неохотно. В погоне за идеальными, стремительными и красивыми моделями, способными не только принести прибыль, но и стать легендарными, неловко признаваться в низком уровне лобового сопротивления.
Поэтому в рейтинг попали те наиболее показательные модели, чьи параметры известны и достоверны.
7. Lada 4×4 или ВАЗ-21213 «Нива»
На фото: Нива 4х4
В этом автомобиле можно найти едва ли не равное количество достоинств и недостатков: проходимость хорошая для одних и недостаточная для других, подвеска достаточно жесткая для преодоления канав и бордюров, но слишком ощутимая во время поездок, особенно для пассажиров заднего ряда, клиренс, размер салона, ремонтопригодность и прочие, и прочие характеристики.
Объективно же автомобиль имеет откровенно слабый 81-сильный двигатель, развивающий не более 137 км/ч.
В немалой степени быстро передвигаться мешают квадратные формы модели, сильно снижающие аэродинамические свойства и столь необходимый для внедорожника большой клиренс, напрямую мешающий борьбе со встречным воздухом.
Некоторые «Кулибины» пытаются исправить ситуацию всевозможными аэродинамическими обвесами, но получаемый результат не всегда сопоставим с расходами на их приобретение и установку, а также потраченным на модернизацию временем.
Поэтому владельцы «Нивы» могут утешиться лишь тем, что ставший культовым «Гелендваген» по аэродинамике проигрывает нашем соотечественнику.
Коэффициент Сх у «Нивы» составляет 0,536.
6. Mercedes-Benz G-класса
Владельцы этого автомобиля грустно шутят, что бронетехника и аэродинамика – понятия не совместимые.
Действительно, модель по-настоящему «страдает» от своей кубической формы, чрезвычайно прожорливого силового агрегата и установленного под острым углом широкого лобового стекла – как бы инженеры ни улучшали силовой агрегат, максимально развиваемая скорость все равно не дотягивает до желаемого и создателями, и автомобилистами уровня.
Даже самая заряженная версия G 65 AMG, получившая под свой капот 630 «лошадей», не в состоянии разогнаться больше чем на 230 км/ч.
При этом разработчики машины в каждом новом поколении меняют, исправляют, модернизируют лишь технические характеристики, оставляя далеко не обтекаемый кузов традиционным, проверенным годами и неизменным.
Коэффициент Сх у Гелендваген составляет 0,54.
5. ВАЗ
Весь ассортимент советской классики не может похвастаться впечатляющей аэродинамикой: лучше всего она у «семерки», а наиболее плачевная – у «шестерки». Эксперты иронично отмечают, что имеющиеся у этих автомобилей показатели полностью соответствуют пособию «Автомобили и тракторы. Основы эргономики и дизайна», составленному МАМИ.
Действительно основательно инженеры подошли уже к ВАЗ 2110, который не только отлично разгонялся и развивал неплохую скорость, но и демонстрировал экономичный расход топлива.
Чтобы повысить аэродинамические показатели, необходимо было полностью изменить конструкцию. Все первые модели имели очень большую лобовую площадь, почти квадратную форму кузова, высокий клиренс и неудачный угол наклона ветрового стекла. Даже вроде бы достаточно обтекаемая на вид «девятка» имеет коэффициент сопротивления немногим меньший, чем его «собратья».
«Шестерка», по сведениям того же учебника, имеет коэффициент 0,54.
А лучшего результата из всего семейства удалось достичь универсалу 2104 с его показателем в 0,53.
Коэффициент Сх самый лучший оказался у ВАЗ 2104 на уровне 0,53 и самый худший у ВАЗ 2106 на уровне 0,56.
4. Hummer h3
Эффектный и мощный внедорожник на трассе не сможет догнать не только импортную малолитражку, но даже российскую Lada Granta.
Этот качественный армейский транспорт создан для преодоления препятствий и покорения любых территорий, а для этого не нужна высокая скорость и обтекаемость кузова.
Трехтонный автомобиль с мощнейшим двигателем V8 6-литрового объема едва разгоняется до 160 км/ч, тогда как даже тольяттинский автопром способен развить 183 км/ч.
Проходимость и ремонтопригодность этого титана заставляют забыть о том, как трудно ему прорываться сквозь толщу воздуха, преодолевая те самые преграды.
Коэффициент Сх у Hammer составляет 0,57.
3. Jeep Wrangler
Городская версия армейского «Виллиса», прошедшая долгий путь от 1941 года до миллениума, категорически не приемлет высокие скорости.
Причем по сравнению с военной машиной современные модели стали еще крупнее, еще длиннее, особенно если рассматривать двухдверное поколение TJ. Эта модель с открытым верхом имеет сопротивление в 0,58.
В 2011 году появилось поколение, получившее более изогнутое лобовое стекло, которое, помимо улучшения аэродинамики, еще и способствовало шумоизоляции.
Дополнительно инженеры и конструкторы изменили некоторые элементы передней части, в том числе бампер, форму капота и радиаторную решетку, что одновременно с повышением коэффициента сопротивления обеспечило пассивной системой безопасности.
Коэффициент Сх у Wrangler составляет от 0,58 (двухверная модификация) до 0,495 у Wrangler Unlimited.
2. УАЗ «Хантер»
Современный внедорожник «Хантер» является практически близнецом УАЗ-469, выпущенного в 1972 году.
Характеристики этого автомобиля можно также увидеть в учебнике МАМИ авторства эксперта по аэродинамике профессора Игоря Степанова. Он сообщает потенциальным владельцам о том, что при 4-метровой длине, 2-метровой ширине, цельнометаллическом кузове и 2-тонном весе маломощные силовые агрегаты с трудом справляются со своей задачей.
Еще большему снижению аэродинамики способствуют сами владельцы, которые страстно любят разного рода тюнинг своих «Охотников», делающий машину эффектнее и брутальнее, но медленнее и неповротливее.
Коэффициент Сх популярного российского внедорожника составляет 0,6
1. Caterham Seven
Парадоксально, что в рейтинг попал спортивный автомобиль, так как эта категория славится как раз обтекаемыми, стремительными формами.
Действительно, эта модель имеет аэродинамику еще более плачевную, чем габаритные и угловатые внедорожники. Этот автомобиль можно назвать обновленной версией легенды автогонок 60-х годов Lotus Seven.
Инженеры и дизайнеры постаралась максимально сохранить исконный облик вплоть до ретроприборов, навесных элементов и сетки вместо радиаторной решетки, а в «сердце» автомобиля — слабый 96-сильный двигатель, разгоняющийся до 161 км/ч, или 260-сильный топовый вариант, доходящий до 250 км/ч.
В совокупности такой дизайн и конструкция позволяют одновременно и быть на вершине антирейтинга по уровне аэродинамики, и показывать неплохие результаты на треке.
Благодарить за такие показатели модель должна предельно сниженный вес, который составляет всего лишь 575 кг. Кроме того, создатели предполагают даже одноместную версию, в которой допустимо убрать пассажирское кресло и тем самым сделать машину еще легче.
Коэффициент Сх у Caterham Seven составляет 0,7
Заключение
Показатели аэродинамических качеств, которые демонстрируют современные модели, показались бы фантастическими буквально десятилетие назад. Коэффициент лобового сопротивления (Cx) объясняет, сколь просто удается автомобилю разрезать воздух, исходя из условного цилиндра, чья площадь поперечного сечения равна максимальной площади сечения конкретной модели.
Для снижения того показателя конструкторы «ужимают» кузов и делают автомобили низкими, придают кузову все более обтекаемую, каплевидную форму, уделяя особой внимание задней части.
Нередко внешне динамичный облик на практике оказывается обманчивым, так как большое значение имеет угол наклона задней части, который должен быть как можно больше.
Таким образом, объяснимо, почему рейтинг самых худших по аэродинамике автомобилей преимущественно составляют внедорожники и универсалы.
Видео про аэродинамику автомобилей в Формуле 1:
Сопротивление транспортного средства (автомобиль)
Рабочие параметры Для проведения эффективных расчетов эксплуатационных характеристик автомобилей должны быть разработаны методы, учитывающие ряд рабочих параметров. Мощность, выдаваемая двигателем, в конечном итоге передается на ведущие колеса в виде тяговой силы. Движению автомобиля по ровной дороге противодействуют сопротивление воздуха и качению. Когда тяговое усилие, сила, доступная в контакте между ведущими колесами и дорогой, больше, чем общее сопротивление на ровной дороге, избыточное тяговое усилие способствует ускорению, преодолению подъемов и тяговому усилию.
Расчет эквивалентного веса, эффективности трансмиссии, положения центра тяжести, устойчивости транспортного средства на уклоне и динамики транспортного средства, движущегося по неровной дороге, также одинаково важны для оценки характеристик транспортного средства. В этой главе обсуждаются и излагаются методы работы с этими темами.
31.1.
Сопротивление транспортного средства – это сила, противодействующая движению и возникающая из-за деформации колеса и земли (последняя пренебрежимо мала для транспортных средств на обычной дороге) и аэродинамических эффектов воздушного потока над поверхностью средство передвижения. Рассмотрено движение транспортного средства для прямолинейного положения, для простоты игнорируя влияние прохождения поворотов.
31.1.1.
Пневматическая шина особенно подходит для использования в дорожных транспортных средствах из-за ее вклада в комфорт, ее превосходных свойств сцепления и потому, что она не разрушает дорожное покрытие до степени более жесткой рулевое колесо.
Однако нагрузка на транспортное средство и тяговое усилие не переносятся без деформации. В случае пневматической шины на твердом покрытии современной дороги деформация шины составляет 90 – 95% сопротивления качению автомобиля. Под сопротивлением качению понимается сила сопротивления транспортного средства, за исключением силы, вызванной аэродинамическими эффектами. Потери на ветер и проскальзывание малы по сравнению с этим. Деформация протектора шины при прохождении через контактную зону приводит к гистерезисным потерям, которые проявляются в виде нагревания и повышения температуры шины.
Сопротивление качению из-за потерь на гистерезис из-за деформированной шины в первую очередь зависит от прогиба шины, вызванного нагрузкой, которую несет шина. Другими параметрами, влияющими на сопротивление качению пневматической шины на твердой поверхности, являются температура шины, давление в шине, скорость транспортного средства, толщина протектора, количество штабелей, состав резины и уровень передаваемого крутящего момента.
Сопротивление качению увеличивается с ростом скорости автомобиля, если все остальные параметры поддерживаются постоянными. Однако на практике увеличение скорости автомобиля приводит к повышению температуры и давления в шинах. Конечным результатом для данной шины является почти постоянное сопротивление качению в зависимости от скорости транспортного средства до тех пор, пока не будет достигнута такая скорость (рис. 31.1), при которой в протекторе вслед за зоной контакта образуется заметная стоячая волна. Сопротивление движению шины в этом состоянии очень быстро возрастает, а энергия, рассеиваемая при деформации, вызванной стоячей волной, способна за очень короткое время разрушить протектор. Поэтому обычно указывается безопасная максимальная скорость для конкретной шины, которая значительно ниже скорости, при которой возникает стоячая волна.
Рис. 31.1. Скорость автомобиля и сопротивление качению.
Поскольку существует прямая зависимость между нагрузкой на шину, прогибом и потерями на гистерезис, а также поскольку вес транспортного средства равен нагрузке на все колеса, сопротивление качению Rr выражается через безразмерный коэффициент качения а, как ,
Rr = aW.
Поскольку сопротивление качению нельзя считать постоянным во всем диапазоне скоростей транспортного средства, обычно добавляют еще один коэффициент b, такой, что
Rr = (a + b V)W, где V — скорость транспортного средства, а W — вес транспортного средства.
Расчеты характеристик автомобиля обычно проводятся при полностью открытой дроссельной заслонке с высоким и довольно постоянным уровнем крутящего момента в нижнем диапазоне скоростей автомобиля, где важно сопротивление качению. Следовательно, используемые коэффициенты сопротивления качению должны соответствовать соответствующему уровню крутящего момента.
31.1.2.
Деформация грунта
Рассматриваются два случая, и оба касаются движения транспортного средства по поверхности, отличной от дороги с твердым покрытием. В первом случае грунт считается «упругим», так что колесо транспортного средства деформирует грунт, который затем возвращается в исходное состояние после проезда транспортного средства.
Во втором случае «пластичным» считается грунт, на котором остается постоянная колея после проезда транспортного средства.
На рис. 31.2 изображен случай жесткого колеса на упругой поверхности. Земля перед движущимся колесом скапливается и течет спереди, под колесом и вокруг него к задней части, что приводит к потере энергии и сопротивлению качению. Усилие этого на колесе состоит в том, чтобы расположить равнодействующую F нормальных сил в некоторой точке A. Вертикальная составляющая равнодействующей силы должна равняться нагрузке W на колесо (из соображений равновесия), а горизонтальная составляющая представляет собой силу сопротивления качению. Съемка моментов относительно центра «О» колеса,
Рис. 31.2. Колесо на упругом грунте.
Если грунт полностью пластичен, энергетический баланс дает
Сопротивление качению = xpz
где x — ширина колеи,
p — нормальное давление между колесом и землей, а
z — глубина возраст утопления.
Чтобы оценить вышеприведенное выражение, необходимо знать нормальное давление (p), которое может быть связано с глубиной погружения (z) как,
Эти три параметра являются свойствами грунта. Это выражение было разработано для случая, когда плоская пластина проникает в пластиковый грунт, но оно также используется для случая колеса транспортного средства.
Используя соотношение, вес транспортного средства = p x предполагаемая площадь контакта под колесами, как p, так и z, и, следовательно, можно найти сопротивление качению, поскольку предполагаемая площадь контакта под колесом является функцией времени погружения, z.
Взаимосвязь между сопротивлением качению и весом транспортного средства является более сложной и требует детальных знаний о грунте или материале грунта в виде трех параметров n, kc и &e-
31.1.3.
Движущееся транспортное средство, вытесняя окружающий воздух, имеет результирующую силу сопротивления, называемую аэродинамическим сопротивлением (просто сопротивление воздуха), и действует на него.
Обычно это сопротивление безразмерно выражается с использованием коэффициента аэродинамического сопротивления Cd-
, где p — плотность воздуха, обычно принимаемая равной 1,23 кг/м3, и скорость транспортного средства Vis (м/с) относительно воздуха.
Для того, чтобы назначить подходящую характеристическую область, необходимо детальное изучение состава аэродинамического сопротивления, которое обусловлено тремя отдельными типами аэродинамических эффектов. (£) Поток воздуха в пограничном слое, приводящий к потере импульса основного
поток. Этот эффект создает сопротивление «трения кожи». (ii) Составляющая от нисходящих вихрей позади транспортного средства, приводящая к индуктивному сопротивлению.
(Hi) Сопротивление «нормального давления», которое можно найти путем интегрирования произведения (нормальное давление x площадь) вокруг транспортного средства. Это создает результирующую силу, противодействующую движению транспортного средства, поскольку разделение потока в задней части транспортного средства приводит к снижению давления на обращенные назад поверхности.
Сопротивление поверхностного трения и индуцированное сопротивление обычно малы по сравнению с сопротивлением нормального давления. Однако сопротивление поверхностного трения может достигать значительных размеров в случае длинного транспортного средства, такого как автобус. Поскольку основной вклад в аэродинамическое сопротивление вносит нормальное сопротивление давлению, соответствующей характеристической площадью является «проекционная лобовая площадь» транспортного средства А.
A = 0,8 (высота транспортного средства над уровнем земли x ширина кузова)
Однако такое приблизительное выражение не является реальной заменой точного измерения, и его следует избегать. Коэффициент аэродинамического сопротивления Cd для конкретного транспортного средства можно считать постоянным, если не учитывать влияние бокового ветра.
Как рассчитать силу аэродинамического сопротивления – x-engineer.org
Содержание
- Определение
- Формула
- Коэффициент аэродинамического сопротивления
- Фронтальная площадь
- Сила аэродинамического сопротивления
- Сила аэродинамического сопротивления
- Пример
- Калькулятор
- Ссылки
Определение
сила .
Тот же принцип применим и к дорожным транспортным средствам: при движении из-за взаимодействия с окружающим воздухом возникает сила сопротивления, которая пытается остановить движение транспортного средства. Эта сила, называемая силой аэродинамического сопротивления, увеличивается по величине со скоростью автомобиля и зависит от формы и размера кузова автомобиля.
Изображение: BMW Vision — Эффективная динамика — Аэродинамика
Авторы и права: BMW
Аэродинамическое сопротивление является важным аспектом конструкции автомобиля, поскольку оно напрямую влияет на потребление энергии и характеристики автомобиля (особенно на высокой скорости).
Назад
Формула
Когда транспортное средство движется по воздуху, перед ним создается динамическое давление, которое в дальнейшем превращается в силу сопротивления. Сила аэродинамического сопротивления , действующая на транспортное средство при движении, аппроксимируется формулой [2]:
F AD = 0,5 · C D · A · ρ AIR · (V — V ВИНД ) 2
(1)
, где:
6.
(1). – сила аэродинамического сопротивления
C d [-] – коэффициент аэродинамического сопротивления
A [м 2 ] – максимальная площадь поперечного сечения автомобиля
ρ воздух [кг/м 3 ] – плотность воздуха ( равно 1,202 кг/м 3 для сухого воздуха при 20 °C и 101,325 кПа)
v [м/с] – скорость автомобиля
v ветер [м/с] – скорость ветра
Если предположить, что ветра нет, уравнение (1) принимает вид: · v 2
(2)
Из уравнения (2) видно, что сила аэродинамического сопротивления увеличивается пропорционально квадрату скорости, поэтому она становится критически важной при более высоких скоростях автомобиля.
Вернуться назад
Коэффициент аэродинамического сопротивления
Коэффициент аэродинамического сопротивления оказывает большое влияние на силу аэродинамического сопротивления. В общем случае коэффициент лобового сопротивления можно определить как аэродинамическое качество формы тела в потоке.
В зависимости от формы кузова коэффициент аэродинамического сопротивления может сильно различаться.
Изображение: Коэффициент аэродинамического сопротивления для различных форм
Чем ниже коэффициент аэродинамического сопротивления, чем ниже сила аэродинамического сопротивления автомобиля, тем выше энергоэффективность.
Оптимальная аэродинамическая форма с меньшим коэффициентом аэродинамического сопротивления — форма капли воды (Cd = 0,04). Из-за трения о воздух капля воды вынуждена принять форму, имеющую наименьшее сопротивление при контакте с воздухом, следовательно, она имеет наименьший коэффициент сопротивления.
Коэффициент аэродинамического сопротивления зависит от типа кузова автомобиля и от частей автомобиля, установленных на внешних поверхностях, таких как: багажник на крыше, брызговики, задний спойлер, боковые зеркала, радиоантенна и стеклоочистители. Эти детали нарушают обтекаемую форму автомобиля и увеличивают его коэффициент аэродинамического сопротивления.
Из [4] Мы можем перечислить коэффициент сопротивления для некоторых общих транспортных средств:
Транспортный| 2005 | 0.400 | |
| Toyota Camry | 1992 | 0.330 |
| Smart Roadster Coupé | 2003 | 0.380 |
| Toyota Prius | 2014 | 0,260 |
| Chevrolet Volt | 2014 | 0,281 |
Go Back 9003.Semend Asred Aselage
33.Semend Aseless Aselage
33.Semend Aseless33.shie Asred Aseless
333.shie Asred Avers Arragic 9008 3. [м 2 ]. Фронтальная площадь автомобиля часто указывается производителем или может быть аппроксимирована с помощью эталонной сетки, как показано на рисунке ниже. Изображение: приблизительная площадь лобовой части автомобиля
Кредит.
Ford Escape Hybrid 2005 1.
080 Toyota Camry 1992 0.703 Smart Roadster Coupé 2003 0.596 Toyota Prius 2014 0.576 Chevrolet Volt 2014 0.622 Go back
Aerodynamic drag force
Using equation (2) and data from the tables above, we может отображать силу аэродинамического сопротивления для транспортных средств со скоростью от 0 до 250 км/ч.
Изображение: Сила аэродинамического сопротивления для нескольких автомобилей
Сила аэродинамического сопротивления увеличивается пропорционально квадрату скорости автомобиля. По этой причине, особенно при высокой скорости автомобиля (> 100 км/ч), аэродинамика имеет решающее значение с точки зрения производительности автомобиля и энергоэффективности.
Если сравнить силу аэродинамического сопротивления с силой сопротивления качению, то можно увидеть, что до 100 км/ч они имеют одинаковое значение.
При более высоких скоростях автомобиля аэродинамические потери намного больше, и они потребляют большую тяговую силу от трансмиссии.
Вернуться назад
Сила аэродинамического сопротивления
Лучший способ понять величину аэродинамического сопротивления — посмотреть на потребляемую мощность для аэродинамических потерь. Сила аэродинамического сопротивления P ad [Вт] рассчитывается путем умножения силы аэродинамического сопротивления F ad [Н] на скорость транспортного средства v [м/с]:
P ad = F ad · v
( 3)
Используя уравнение (3), мы можем построить график зависимости мощности аэродинамического сопротивления от скорости транспортного средства, например, выше:
Изображение: Мощность аэродинамического сопротивления для нескольких транспортных средств
сопротивление воздуха значительное. Во многих случаях максимальная скорость автомобиля ограничивается аэродинамическим сопротивлением, так как оно потребляет большую часть мощности колеса и нет запаса мощности для разгона.
Назад
Пример
Рассчитайте силу и мощность аэродинамического сопротивления для автомобиля с коэффициентом сопротивления 0,4 и площадью лобового сечения 1,08 м 2 , движущегося со скоростью 100 км/ч.
Шаг 1 . Преобразование скорости автомобиля из км/ч в м/с.
v = 100/3,6 = 27,78 м/с
Шаг 2 . Рассчитайте силу аэродинамического сопротивления, используя уравнение (2).
F ad = 0,5 · 0,4 · 1,08 · 1,202 · 27,78 2 = 200 Н
Шаг 3 . Рассчитайте мощность аэродинамического сопротивления, используя уравнение (3).
P AD = 200 · 27,78 = 5556 W = 5,556 кВт
Go Back
Калькулятор
C D [-] = A. 4 [M 4] [4] [. кг/м 3 ] = В [км/ч] = F ad [N] = P ad [кВт]0203
Назад
Ссылки
[1] Хеннинг Валлентовиц, Продольная динамика транспортных средств – лекция, IKA RWTH, Ахен, 2004.
[2] Ларс Эрикссон, Ларс Нильсен, Моделирование и управление двигателями, Wiley, 2014.
[3] Automotive Handbook, 9th Edition, Bosch, 2014.
[4] https://en.wikipedia.org/wiki/Automobile_drag_coefficient
[5] https://www.motorrend.com/how -to/hdrp-0609-aero-tricks-tips
№ 1520: коэффициент аэродинамического сопротивления автомобиля
Сегодня аэродинамика и автомобили. Университет
Инженерного колледжа Хьюстона представляет этот
сериал о машинах, которые делают наши
цивилизация управляется, и люди, чья изобретательность
создал их.
Первые автомобили были сделаны
до того, как братья Райт полетели и до того, как они
даже начали проводить испытания в аэродинамической трубе. Аэродинамические трубы
использовались в конструкции самолетов с самого начала.
Автомобиль
конструкторы были гораздо медленнее, чтобы увидеть, что аэродинамика
также пострадало их работают.
На рубеже ХХ века Германия установила
создание ряда технических университетов высокого уровня,
что вывело их далеко вперед в аэродинамических исследованиях.
В 1921 году завод Zeppelin Airship Works
впервые изучил обтекаемость автомобиля на ветру
туннели.
Автомобильный тормоз может быть серьезным пожирателем бензина. Когда я
был ребенком, я развлекался в долгих автомобильных поездках
высунув руку из окна и повернув ее в
под разными углами к ветру. Силы, даже на
маленькая детская рука, были довольно сильными. И маленький
изменения формы и ориентации моей руки
внесли огромные различия.
Обычной мерой аэродинамической эффективности является
коэффициент аэродинамического сопротивления, CD . Это
сравнивает силу сопротивления на любой скорости с
сила, необходимая, чтобы остановить весь воздух перед
автомобиль. Коэффициенты лобового сопротивления для первых квадратных автомобилей
были выше 0,7. Вместо того, чтобы позволить воздуху ускользнуть
прошлого, они остановили большую часть этого.
В большинстве современных автомобилей эта цифра снижена до
мало 0,3. Конечно, это нечто большее, чем просто
снижение сопротивления. Достаточно легко уменьшить сопротивление
если мы позволим себе создать другие аэродинамические
проблемы, когда мы делаем. Автомобиль должен быть рассчитан на
отрицательный подъем.
Ветер должен прижимать его основательно
вниз против дороги. И машин не должно быть
уязвимы для бокового ветра.
Уже в 1907 году обтекаемый гоночный автомобиль под названием
Ракета достигла скорости 132 мили в час раньше
он стал воздушным. У него был низкий коэффициент аэродинамического сопротивления,
но это был еще плохой аэродинамический дизайн.
Оптимизация была новой
иконой дизайна 1930-х годов, но это было скорее иллюзия скорости, чем реальное снижение лобового сопротивления.
Только несколько автомобилей прошли испытания в аэродинамической трубе.
знаменитый Chrysler
Airflow был исключением, с гораздо меньшим
сопротивление, чем большинство автомобилей, следующих за ним.
Инженерам потребовалось время, чтобы понять, что они должны
сгладить днище автомобиля настолько, насколько
Топ. Потребовалось время, чтобы увидеть, что острые углы на
перед автомобилем были ужасные тормоза. Только в
в прошлом поколении 18-колесные машины дали ростки
странные, но эффективные, уменьшающие лобовое сопротивление капоты над
их такси. И только со времён ВМВ есть аэродинамическая труба
испытания были обычной частью дизайна автомобиля. Только
недавно было точное компьютерное моделирование пусть
инженеры используют быстрые пробы и ошибки, чтобы улучшить
аэродинамические конструкции.
Вот почему сегодняшние автомобили обладают таким малым лобовым сопротивлением. А также
конечно, они все больше и больше похожи друг на друга.
В качестве
дизайнеры работают с растущим знанием дизайна
ограничений, они приближаются к оптимальным конструкциям, которые
не может сильно отличаться от одной машины к другой. Это
просто потому, что наши машины такие прекрасные машины, что
мы больше не смотрим на дикий солнечный свет
возможности, которые привели нас туда, где мы есть.
Я Джон Линхард из Хьюстонского университета.
где нас интересует, как изобретательные умы
Работа.
(Музыкальная тема)
Сопротивление воздуху: невидимый враг в конструкции транспортных средств
Разработчики транспортных средств, независимо от того, специализируются ли они на спортивных автомобилях, грузовиках, обычных транспортных средствах или даже мотоциклах, каждый день сталкиваются с невидимым врагом — сопротивлением воздуха.
Проще говоря, когда тело движется, воздух вокруг него создает сопротивление в направлении, противоположном движению. Для транспортных средств сопротивление воздуха влияет на комфорт пассажиров, расход топлива, устойчивость и многие другие факторы производительности.
Аэродинамика
Автомобильный
Формула 1
Поднимите и перетащите
Сопротивление воздуха
Измерение сопротивления воздуха Источник: TheOtherJesse, Wikimedia Commons Он состоит из плотности жидкости (которую мы не можем изменить), контрольной площади (площадь лобовой части в случае автомобилей или мотоциклов), коэффициента лобового сопротивления (определяемого формой тела) и скорости потока ( относительно объекта)
Уравнение показывает, что единственные параметры, которые дизайнер может изменить, – это опорная площадь объекта и коэффициент сопротивления. Для автомобилей и многих других объектов эталонной площадью является проекция передней части транспортного средства.
Коэффициент аэродинамического сопротивления варьируется от 0, а более низкий коэффициент аэродинамического сопротивления указывает на то, что транспортное средство будет менее аэродинамическим, что снижает коэффициент аэродинамического сопротивления и улучшает характеристики транспортного средства в отношении скорости и топливной экономичности. Это два значения, на которых необходимо сосредоточиться, чтобы уменьшить аэродинамическое сопротивление автомобиля.
Генеральный директор SimScale Дэвид Хейни тестирует возможности облачного моделирования для решения инженерной задачи. Заполните форму и посмотрите бесплатную запись вебинара, чтобы узнать больше!
Расчет сопротивления воздуха
Эволюция конструкции обтекаемых транспортных средств Геометрия автомобиля по отношению к сопротивлению воздуха (Источник: Эшаан, 1992 г., из Wikimedia Commons) как правило, довольно «квадратный», с угловатыми формами.
С тех пор многое изменилось: производители автомобилей постоянно улучшают аэродинамику, стремясь сделать каждую новую модель более плавной и «скользкой», чем предыдущая, позволяя воздуху легко обтекать ее с наименьшим возможным сопротивлением. Современные методы проектирования транспортных средств, которые помогают уменьшить сопротивление воздуха, включают в себя, помимо сглаживания общей формы автомобиля, углубление дворников и дверных ручек, обтекаемость наружных зеркал, устранение выступающих желобов по краям крыши и многое другое — все из которых помогают уменьшить сопротивление и предотвратить потерю эффективности.
Аэродинамические трубы и аэродинамическое сопротивление
Использование аэродинамических труб в конструкции транспортных средств Однако это относительно недавние разработки. До 19В 80-х годах обтекаемая конструкция транспортных средств была ограничена гонками и спортивными автомобилями высокого класса, пока не были введены испытания в аэродинамической трубе, которые вывели на рынок потребительские автомобили с оптимизированной аэродинамикой.
Вскоре аэродинамические трубы стали одним из важнейших инструментов улучшения аэродинамики транспортных средств.
В аэродинамической трубе прототип транспортного средства фиксируется на месте, когда к нему направляется поток воздуха, чтобы имитировать воздушный поток, с которым транспортное средство столкнулось бы при движении по реальной дороге. Затем измеряется величина создаваемого сопротивления, чтобы оценить коэффициент сопротивления и оценить общие аэродинамические характеристики транспортного средства.
Улучшение аэродинамического сопротивления
Улучшение аэродинамики конструкции автомобиля с помощью моделирования Анализ аэродинамики автомобиля Perrinn F1 с помощью SimScale Физические аэродинамические трубы были отраслевым стандартом для производителей и конструкторов автомобилей до введения моделирования потоков жидкости с помощью виртуальных аэродинамических труб. Помимо высоких затрат на первоначальную настройку, физические испытания в аэродинамической трубе отнимают много времени и могут значительно увеличить цикл проектирования продукта.
С другой стороны, инструменты вычислительной гидродинамики (CFD) значительно более эффективны и значительно сокращают затраты на проектирование и время, позволяя инженерам тестировать свои конструкции в гораздо большем разнообразии условий эксплуатации.
Чтобы проиллюстрировать применение испытаний в виртуальной аэродинамической трубе при внешнем аэродинамическом анализе транспортных средств, мы выбрали несколько проектов моделирования из публичной библиотеки проектов SimScale. Наши инженеры-симуляторы вместе с нашим активным сообществом пользователей выполнили многочисленные симуляции аэродинамики транспортных средств, включая грузовики, спортивные автомобили, футуристические автомобили, автомобили F1, гоночные автомобили FSAE, LMP1, а также самолеты, гоночный бобслей и другие разные проекты. .
В автомобильной промышленности наиболее целенаправленное применение оптимизации аэродинамики автомобиля можно найти в Формуле 1. С конца 60-х годов инженеры Формулы 1 работали над аэродинамикой своих автомобилей с двойной целью: минимизировать аэродинамическое сопротивление и максимизировать прижимная сила.
Для достижения обеих целей инженеры создали множество различных и экстравагантных решений. Например, конструкция Brabham BT46B, которая создавала высокий уровень прижимной силы с помощью вентилятора, не только усиливала охлаждение, но и вытягивала воздух из-под автомобиля.
Сопротивление воздуха
Заключение Минимизация сопротивления воздуха и оптимизация аэродинамических характеристик остается одной из ключевых задач для конструкторов в автомобильной промышленности. Это больше не является исключительной прерогативой инженеров гоночных автомобилей и оказывает реально ощутимое влияние на различные аспекты характеристик автомобилей потребительского класса, включая расход топлива, комфорт пассажиров и многое другое. Однако нельзя отрицать, что появление инструментов виртуального прототипирования и моделирования значительно упростило решение этой проблемы, чем раньше. Если вы хотите узнать больше о том, как свести к минимуму сопротивление воздуха с помощью инженерного моделирования, запустите бесплатную пробную версию SimScale и используйте возможности платформы CFD для оптимизации собственных проектов.
Чтобы узнать, как использовать SimScale, посмотрите вебинар «Как оптимизировать гоночные автомобили с помощью облачной CFD», организованный совместно с журналом Racecar Engineering. Просто заполните эту короткую форму, и она будет воспроизводиться автоматически.
Взгляд на самые аэродинамические автомобили из когда-либо созданных
Бензиновые, дизельные или электрические автопроизводители прилагают все усилия, чтобы выжать из своих автомобилей все до последней капли. Большая часть этих усилий направлена на оптимизацию аэродинамики. Уменьшение лобового сопротивления является основным направлением деятельности инженеров, работающих над новейшими высокоэффективными моделями, и за эти годы породило множество инновационных разработок. Мы рассмотрим, почему снижение лобового сопротивления так важно, а также некоторые уникальные транспортные средства, появившиеся в результате этих усилий по оптимизации.
Boo To Air Resistance
График, показывающий увеличение аэродинамического сопротивления и сопротивления качению при увеличении скорости.
Обратите внимание на гораздо более высокий вклад аэродинамического сопротивления, особенно на скоростях шоссе. Независимо от того, ищете ли вы более низкую экономию топлива или просто пытаетесь проехать как можно больше миль от вашей батареи, дрейф — ваш враг. Толкание автомобиля по воздуху требует работы, и чем быстрее вы едете, тем сильнее отталкивает воздух. К сожалению, сопротивление пропорционально квадрату скорости, поэтому при удвоении скорости сила сопротивления увеличивается в четыре раза. При скорости выше примерно 20 км/ч (12,4 мили в час) аэродинамическое сопротивление является самой большой силой, воздействующей на автомобиль, затмевая сопротивление качению по мере увеличения скорости.
Разумеется, можно принять меры для уменьшения этого сопротивления. Помогает создание автомобиля с более гладким профилем, который деликатно разделяет воздух спереди и позволяет ему мягко воссоединяться сзади. Помогает уменьшение размера и количества выступов, а также уменьшение общей лобовой площади автомобиля.
При внимательном отношении к этим факторам автопроизводители могут значительно снизить лобовое сопротивление с сопутствующим повышением эффективности.
О скользкости автомобиля часто говорят с точки зрения коэффициента аэродинамического сопротивления или С д . Это безразмерный коэффициент, который количественно определяет величину сопротивления, создаваемого данным объектом при прохождении через жидкость, такую как вода или воздух. В некоторых анализах также важно учитывать C d A — коэффициент аэродинамического сопротивления, умноженный на лобовую площадь автомобиля. Два автомобиля могут иметь одинаковую обтекаемую форму, но если один больше другого, он, естественно, будет испытывать большее сопротивление.
Коэффициенты сопротивления различных основных форм. Обратите внимание, что важно то, как воздух возвращается вокруг объекта, а не только фронтальный профиль. В качестве ориентира, плоская пластина, пытающаяся проложить себе путь через воздух, выдаст C d 1,28, в то время как пуля с дозвуковой скоростью может пройти 0,295.
Типичные современные седаны и купе имеют коэффициент аэродинамического сопротивления от 0,25 до 0,3, а внедорожники часто имеют более высокие значения от 0,35 до 0,45 из-за их более высокой и более квадратной конструкции. Спорткары, построенные с упором на прижимную силу, естественно, имеют более высокие значения C d из-за индуктивного сопротивления аэродинамических элементов.
1999 Honda Insight, ранний гибридный автомобиль, оказался в нижней части этого диапазона, заявив, что C d равен 0,25, что в то время считалось лучшим в своем классе. Однако новые конкуренты в этом пространстве еще больше улучшили это. Mercedez Benz S 350 BlueTec получил 0,24, как и Tesla Model S при запуске в 2012 году. утверждая цифру всего 0,208. Однако последний Mercedes EQS превосходит оба пункта с показателем всего 0,2.
Дело не только в том, как вы выглядите
Интересно отметить, что в то время как ранние гибриды 1990-х годов имели явно стремительный и обтекаемый дизайн, современные автомобили превзошли эти цифры, не прибегая к таким бесформенным яйцевидным формам.
Часто аэродинамические выгоды можно получить, тщательно формируя поток тонкими способами, а не сосредотачиваясь на макроформе автомобиля в целом. Другие выгоды могут быть получены благодаря техническому прогрессу; Электромобили избавились от больших радиаторов спереди и, таким образом, имеют гораздо более обтекаемые бамперы.
Обтекаемый дизайн характерен для концептуальных автомобилей, ориентированных на эффективность, таких как GM EV1 и VW XL1. Закрытые задние колеса — один из наиболее распространенных способов уменьшить очевидные источники лобового сопротивления, хотя и за счет простой замены шин. Однако серийные автомобили, естественно, ограничены в выборе дизайна, что вынуждает автопроизводителей идти на компромисс, когда речь идет об обтекаемости. Некоторые оптимизации просты, например, замена штыревых антенн на низкопрофильные акульи плавники или добавление аэродинамических покрытий на колеса. С другими сложнее — правила гласят, что боковые зеркала являются обязательными в большинстве юрисдикций, в то время как многие автопроизводители настаивают на принятии камер, чтобы сбрить выступы, чтобы минимизировать сопротивление.
Даже кажущиеся незначительными правила, такие как расстояние от фары до земли или высота капота, могут оказать серьезное влияние на дизайн. Ожидания клиентов в отношении комфорта салона и места для багажа также могут быть проблемой. Таким образом, некоторые из самых низких показателей лобового сопротивления были получены от экспериментальных концептуальных автомобилей. General Motors EV1 1996 года отличается невероятно низким значением C d , составляющим всего 0,19. Это была попытка GM создать настоящий, пригодный для массового использования электромобиль. Автомобиль привлек ярых фанатов среди участников программы ограниченной аренды, но его ущемляли ограниченный запас хода и двухместный салон. Автомобили были отозваны по истечении срока аренды, и подавляющее большинство из них было раздавлено. Точно так же Volkswagen XL1 соответствовал EV1 C d с показателем 0,19 после выпуска в 2013 году. Он был разработан в соответствии с жестким заданием председателя правления Фредерика Пиеха, чтобы выжать 100 км из одного литра дизельного топлива.
Серийная версия, оснащенная двухцилиндровым двигателем мощностью 35 кВт и электродвигателем мощностью 20 кВт, развивала 0,9л/100км в реальных условиях. Ограниченный производственным циклом всего в 200 единиц, автомобиль не имел боковых зеркал или зеркал заднего вида, а также заднего ветрового стекла. Пассажиры сидят тандемом, один за другим, а не бок о бок, чтобы минимизировать фронтальную площадь для максимальной эффективности. Более того, транспортные средства, подобные тем, которые участвовали в World Solar Challenge, разработаны для оптимальной производительности, позволяющей максимально использовать их ограниченную солнечную энергию. Запись Sunraycer 1987 года отличалась обтекаемым кузовом с изображением C 9.0125 d всего 0,125, что вынуждает водителя лежать в машине почти на спине. Аналогичным образом, участие в Эко-марафоне Shell следует примерно той же философии: Eco-runner 8 2018 года набирает скользкие 0,045.
1930-е годы были прекрасным временем для этого парящего взгляда
Schlörwagen (также известный как «геттингерское яйцо») был проектной концепцией, намного опередившей свое время, основанной на шасси Mercedes с задним расположением двигателя и построенной в 1930-х годах.
Однако история оптимизации автомобилей началась намного раньше, чем после 19-го века.Топливный кризис 73 года привел к тому, что американцы начали массово покупать компактные автомобили. Основные аэродинамические концепции, заставляющие объекты скользить по воздуху, применялись давно, когда повальное увлечение модернизацией 1930-х годов коснулось всего, от поездов до автомобилей и тостеров. Tatra T77A был одним из первых автомобилей, разработанных с упором на аэродинамику, но были реализованы и более совершенные конструкции.
Пожалуй, самым экстремальным дизайном того времени был автомобиль, известный как Schlörwagen, названный в честь его конструктора Карла Шлера. Прототип, построенный на шасси Mercedes с задним расположением двигателя, как сообщается, разместил C 9.0125 д 0,15. Это было достигнуто за счет выбора дизайна, который в то время считался диким; весь автомобиль имел форму единого гладкого яйца с минимальными выступами, за что получил прозвище «геттингенское яйцо».
Он полностью закрывал не только задние колеса, но и передние, что требовало кузова шириной 2,10 м, что для того времени считалось смехотворно большим. Окна установлены максимально заподлицо, чтобы уменьшить любые помехи для воздуха, придавая автомобилю футуристический вид, намного опередивший свое время. Однако серийный выпуск автомобиля всерьез не рассматривался, несмотря на впечатляющий дизайн.
В целом, мы, вероятно, увидим, что будущие модели крупных автопроизводителей продолжат тенденцию к снижению показателей лобового сопротивления, поскольку битва за пробег в пространстве электромобилей накаляется. Многие выгоды все еще остаются на столе, поскольку регулирующие органы медленно продвигают правила, касающиеся зеркал и других технологий, которые могут еще больше улучшить цифры. При этом потребители будут по-прежнему требовать минимальных стандартов комфорта, пространства и безопасности, а это означает, что в ближайшее время мы вряд ли будем ездить в заостренных слезинках.
аэродинамика — Является ли коэффициент аэродинамического сопротивления/скорость транспортного средства основанным на лобовой площади?
$\begingroup$
Определяет ли площадь лобовой части автомобиля коэффициент скорости/лобового сопротивления автомобиля?
У Ferrari 458 Cd 0,33, а у Corvette C6 Cd 0,29.
Означает ли это, что у Corvette будет меньше лобовая площадь и больше скорость, а у Ferrari должна быть больше прижимной силы?
- аэродинамика
- сопротивление
$\endgroup$
92A}$
где:
$F_d$ — сила сопротивления, которая по определению является составляющей силы в направлении скорости потока;
$\rho$ — массовая плотность жидкости;
$u$ — скорость потока объекта относительно жидкости;
$A$ — это область отсчета .
Ответ на ваш первый вопрос: » Да, эталонной площадью для автомобилей и многих других объектов является проекция лобовой площади транспортного средства «. Однако коэффициент сопротивления обычно определяется экспериментально, и известные экспериментальные результаты показывают, что он также зависит от формы объектов, как показано ниже, при условии, что эталонная площадь постоянна для всех случаев:
Должно быть отметил, что «форма» также влияет на «силу сопротивления», в свою очередь, на «тягу вниз», поскольку $F_d = f(C_d, \rho, u, A)$ и $u = f(C_d, \rho , A, F_d)$ Таким образом, из-за сложных взаимозависимостей нет простого ответа на ваши последние вопросы.
0011
$\endgroup$
$\begingroup$
Нет Cd не имеет прямого отношения к лобовой области, он связан с тем, насколько эта лобовая область, движущаяся против потока воздуха, должна бороться с сопротивлением.
Например, рассмотрим прицепы старых полуприцепов. Поскольку они были квадратными, у них был большой Cd и требовалось много топлива даже для небольшой лобовой площади.
В современных прицепах они имеют закругленные углы и регулируемые юбки прицепа на конце, чтобы сделать их более обтекаемыми.
Таким образом, современный прицеп с большей передней площадью имеет меньший Cd, создает меньшее лобовое сопротивление и намного более экономичен.
Если мы возьмем два одинаковых кубика и сравним их, они будут иметь одинаковые Cd. Теперь, если вы добавите полусферу к передней и задней части одного из кубов, его Cd упадет почти до половины другого куба.
$\endgroup$
$\begingroup$
$C_D$ — это коэффициент удобства, используемый инженерами для упрощения гидромеханики автомобилей и аэродинамики.
Упрощенное объяснение состоит в том, что $C_D$ нормализует аэродинамическое сопротивление к лобовой области , чтобы было легче сравнивать разные транспортные средства.
Заявление
«Определяет ли лобовая площадь транспортного средства коэффициент скорость/лобовое сопротивление автомобиля?»
для меня означает, что размер лобной области влияет на $C_D$ (это означает, что чем больше площадь, тем больше $C_D$ или наоборот. Однако $C_D$ в значительной степени не зависит от лобной площади, хотя он рассчитывается из него.92 A}$$
где:
- Аэродинамическая сила $Drag$
- $\rho$ плотность воздуха
- $u$ это скорость
- $A$ — лобовая область.
Так, например, изменение углов заднего спойлера практически не влияет на переднюю часть, но может значительно изменить $C_D$. В этом отношении эти двое независимы.
Единственный случай, о котором я могу думать (возможно, есть и другие), когда $C_D$ зависит от лобовой площади, это когда лобная площадь становится слишком маленькой, и поэтому число Рейнольдса значительно изменяется.
Изображение: приблизительная площадь лобовой части автомобиля
Кредит.
080Go back
Aerodynamic drag force
Using equation (2) and data from the tables above, we может отображать силу аэродинамического сопротивления для транспортных средств со скоростью от 0 до 250 км/ч.
Изображение: Сила аэродинамического сопротивления для нескольких автомобилей
Сила аэродинамического сопротивления увеличивается пропорционально квадрату скорости автомобиля. По этой причине, особенно при высокой скорости автомобиля (> 100 км/ч), аэродинамика имеет решающее значение с точки зрения производительности автомобиля и энергоэффективности.
Если сравнить силу аэродинамического сопротивления с силой сопротивления качению, то можно увидеть, что до 100 км/ч они имеют одинаковое значение.
При более высоких скоростях автомобиля аэродинамические потери намного больше, и они потребляют большую тяговую силу от трансмиссии.
Вернуться назад
Сила аэродинамического сопротивления
Лучший способ понять величину аэродинамического сопротивления — посмотреть на потребляемую мощность для аэродинамических потерь. Сила аэродинамического сопротивления P ad [Вт] рассчитывается путем умножения силы аэродинамического сопротивления F ad [Н] на скорость транспортного средства v [м/с]:
P ad = F ad · v
( 3)
Используя уравнение (3), мы можем построить график зависимости мощности аэродинамического сопротивления от скорости транспортного средства, например, выше:
Изображение: Мощность аэродинамического сопротивления для нескольких транспортных средств
сопротивление воздуха значительное. Во многих случаях максимальная скорость автомобиля ограничивается аэродинамическим сопротивлением, так как оно потребляет большую часть мощности колеса и нет запаса мощности для разгона.
Назад
Пример
Рассчитайте силу и мощность аэродинамического сопротивления для автомобиля с коэффициентом сопротивления 0,4 и площадью лобового сечения 1,08 м 2 , движущегося со скоростью 100 км/ч.
Шаг 1 . Преобразование скорости автомобиля из км/ч в м/с.
v = 100/3,6 = 27,78 м/с
Шаг 2 . Рассчитайте силу аэродинамического сопротивления, используя уравнение (2).
F ad = 0,5 · 0,4 · 1,08 · 1,202 · 27,78 2 = 200 Н
Шаг 3 . Рассчитайте мощность аэродинамического сопротивления, используя уравнение (3).
P AD = 200 · 27,78 = 5556 W = 5,556 кВт
Go Back
Калькулятор
| C D [-] = | A. 4 [M 4] [4] [. кг/м 3 ] = | В [км/ч] = | |
| F ad [N] = | P ad [кВт]0203 | ||
Назад
Ссылки
[1] Хеннинг Валлентовиц, Продольная динамика транспортных средств – лекция, IKA RWTH, Ахен, 2004.
[2] Ларс Эрикссон, Ларс Нильсен, Моделирование и управление двигателями, Wiley, 2014.
[3] Automotive Handbook, 9th Edition, Bosch, 2014.
[4] https://en.wikipedia.org/wiki/Automobile_drag_coefficient
[5] https://www.motorrend.com/how -to/hdrp-0609-aero-tricks-tips
№ 1520: коэффициент аэродинамического сопротивления автомобиля
Сегодня аэродинамика и автомобили. Университет Инженерного колледжа Хьюстона представляет этот сериал о машинах, которые делают наши цивилизация управляется, и люди, чья изобретательность создал их.
Первые автомобили были сделаны
до того, как братья Райт полетели и до того, как они
даже начали проводить испытания в аэродинамической трубе. Аэродинамические трубы
использовались в конструкции самолетов с самого начала.
Автомобиль
конструкторы были гораздо медленнее, чтобы увидеть, что аэродинамика
также пострадало их работают.
На рубеже ХХ века Германия установила создание ряда технических университетов высокого уровня, что вывело их далеко вперед в аэродинамических исследованиях. В 1921 году завод Zeppelin Airship Works впервые изучил обтекаемость автомобиля на ветру туннели.
Автомобильный тормоз может быть серьезным пожирателем бензина. Когда я
был ребенком, я развлекался в долгих автомобильных поездках
высунув руку из окна и повернув ее в
под разными углами к ветру. Силы, даже на
маленькая детская рука, были довольно сильными. И маленький
изменения формы и ориентации моей руки
внесли огромные различия.
Обычной мерой аэродинамической эффективности является коэффициент аэродинамического сопротивления, CD . Это сравнивает силу сопротивления на любой скорости с сила, необходимая, чтобы остановить весь воздух перед автомобиль. Коэффициенты лобового сопротивления для первых квадратных автомобилей были выше 0,7. Вместо того, чтобы позволить воздуху ускользнуть прошлого, они остановили большую часть этого.
В большинстве современных автомобилей эта цифра снижена до
мало 0,3. Конечно, это нечто большее, чем просто
снижение сопротивления. Достаточно легко уменьшить сопротивление
если мы позволим себе создать другие аэродинамические
проблемы, когда мы делаем. Автомобиль должен быть рассчитан на
отрицательный подъем.
Ветер должен прижимать его основательно
вниз против дороги. И машин не должно быть
уязвимы для бокового ветра.
Уже в 1907 году обтекаемый гоночный автомобиль под названием Ракета достигла скорости 132 мили в час раньше он стал воздушным. У него был низкий коэффициент аэродинамического сопротивления, но это был еще плохой аэродинамический дизайн.
Оптимизация была новой
иконой дизайна 1930-х годов, но это было скорее иллюзия скорости, чем реальное снижение лобового сопротивления.
Только несколько автомобилей прошли испытания в аэродинамической трубе.
знаменитый Chrysler
Airflow был исключением, с гораздо меньшим
сопротивление, чем большинство автомобилей, следующих за ним.
Инженерам потребовалось время, чтобы понять, что они должны сгладить днище автомобиля настолько, насколько Топ. Потребовалось время, чтобы увидеть, что острые углы на перед автомобилем были ужасные тормоза. Только в в прошлом поколении 18-колесные машины дали ростки странные, но эффективные, уменьшающие лобовое сопротивление капоты над их такси. И только со времён ВМВ есть аэродинамическая труба испытания были обычной частью дизайна автомобиля. Только недавно было точное компьютерное моделирование пусть инженеры используют быстрые пробы и ошибки, чтобы улучшить аэродинамические конструкции.
Вот почему сегодняшние автомобили обладают таким малым лобовым сопротивлением. А также
конечно, они все больше и больше похожи друг на друга.
В качестве
дизайнеры работают с растущим знанием дизайна
ограничений, они приближаются к оптимальным конструкциям, которые
не может сильно отличаться от одной машины к другой. Это
просто потому, что наши машины такие прекрасные машины, что
мы больше не смотрим на дикий солнечный свет
возможности, которые привели нас туда, где мы есть.
Я Джон Линхард из Хьюстонского университета. где нас интересует, как изобретательные умы Работа.
Сопротивление воздуху: невидимый враг в конструкции транспортных средств
Разработчики транспортных средств, независимо от того, специализируются ли они на спортивных автомобилях, грузовиках, обычных транспортных средствах или даже мотоциклах, каждый день сталкиваются с невидимым врагом — сопротивлением воздуха.
Проще говоря, когда тело движется, воздух вокруг него создает сопротивление в направлении, противоположном движению. Для транспортных средств сопротивление воздуха влияет на комфорт пассажиров, расход топлива, устойчивость и многие другие факторы производительности.
Аэродинамика Автомобильный Формула 1 Поднимите и перетащите
Сопротивление воздуха
Измерение сопротивления воздухаИсточник: TheOtherJesse, Wikimedia Commons Он состоит из плотности жидкости (которую мы не можем изменить), контрольной площади (площадь лобовой части в случае автомобилей или мотоциклов), коэффициента лобового сопротивления (определяемого формой тела) и скорости потока ( относительно объекта)
Уравнение показывает, что единственные параметры, которые дизайнер может изменить, – это опорная площадь объекта и коэффициент сопротивления. Для автомобилей и многих других объектов эталонной площадью является проекция передней части транспортного средства.
Коэффициент аэродинамического сопротивления варьируется от 0, а более низкий коэффициент аэродинамического сопротивления указывает на то, что транспортное средство будет менее аэродинамическим, что снижает коэффициент аэродинамического сопротивления и улучшает характеристики транспортного средства в отношении скорости и топливной экономичности. Это два значения, на которых необходимо сосредоточиться, чтобы уменьшить аэродинамическое сопротивление автомобиля.
Генеральный директор SimScale Дэвид Хейни тестирует возможности облачного моделирования для решения инженерной задачи. Заполните форму и посмотрите бесплатную запись вебинара, чтобы узнать больше!
Расчет сопротивления воздуха
Эволюция конструкции обтекаемых транспортных средств Геометрия автомобиля по отношению к сопротивлению воздуха (Источник: Эшаан, 1992 г., из Wikimedia Commons) как правило, довольно «квадратный», с угловатыми формами.
С тех пор многое изменилось: производители автомобилей постоянно улучшают аэродинамику, стремясь сделать каждую новую модель более плавной и «скользкой», чем предыдущая, позволяя воздуху легко обтекать ее с наименьшим возможным сопротивлением.Современные методы проектирования транспортных средств, которые помогают уменьшить сопротивление воздуха, включают в себя, помимо сглаживания общей формы автомобиля, углубление дворников и дверных ручек, обтекаемость наружных зеркал, устранение выступающих желобов по краям крыши и многое другое — все из которых помогают уменьшить сопротивление и предотвратить потерю эффективности.
Аэродинамические трубы и аэродинамическое сопротивление
Использование аэродинамических труб в конструкции транспортных средств Однако это относительно недавние разработки. До 19В 80-х годах обтекаемая конструкция транспортных средств была ограничена гонками и спортивными автомобилями высокого класса, пока не были введены испытания в аэродинамической трубе, которые вывели на рынок потребительские автомобили с оптимизированной аэродинамикой.
Вскоре аэродинамические трубы стали одним из важнейших инструментов улучшения аэродинамики транспортных средств.
В аэродинамической трубе прототип транспортного средства фиксируется на месте, когда к нему направляется поток воздуха, чтобы имитировать воздушный поток, с которым транспортное средство столкнулось бы при движении по реальной дороге. Затем измеряется величина создаваемого сопротивления, чтобы оценить коэффициент сопротивления и оценить общие аэродинамические характеристики транспортного средства.
Улучшение аэродинамического сопротивления
Улучшение аэродинамики конструкции автомобиля с помощью моделирования Анализ аэродинамики автомобиля Perrinn F1 с помощью SimScale Физические аэродинамические трубы были отраслевым стандартом для производителей и конструкторов автомобилей до введения моделирования потоков жидкости с помощью виртуальных аэродинамических труб. Помимо высоких затрат на первоначальную настройку, физические испытания в аэродинамической трубе отнимают много времени и могут значительно увеличить цикл проектирования продукта.
С другой стороны, инструменты вычислительной гидродинамики (CFD) значительно более эффективны и значительно сокращают затраты на проектирование и время, позволяя инженерам тестировать свои конструкции в гораздо большем разнообразии условий эксплуатации.
Чтобы проиллюстрировать применение испытаний в виртуальной аэродинамической трубе при внешнем аэродинамическом анализе транспортных средств, мы выбрали несколько проектов моделирования из публичной библиотеки проектов SimScale. Наши инженеры-симуляторы вместе с нашим активным сообществом пользователей выполнили многочисленные симуляции аэродинамики транспортных средств, включая грузовики, спортивные автомобили, футуристические автомобили, автомобили F1, гоночные автомобили FSAE, LMP1, а также самолеты, гоночный бобслей и другие разные проекты. .
В автомобильной промышленности наиболее целенаправленное применение оптимизации аэродинамики автомобиля можно найти в Формуле 1. С конца 60-х годов инженеры Формулы 1 работали над аэродинамикой своих автомобилей с двойной целью: минимизировать аэродинамическое сопротивление и максимизировать прижимная сила.
Для достижения обеих целей инженеры создали множество различных и экстравагантных решений. Например, конструкция Brabham BT46B, которая создавала высокий уровень прижимной силы с помощью вентилятора, не только усиливала охлаждение, но и вытягивала воздух из-под автомобиля.
Сопротивление воздуха
Заключение Минимизация сопротивления воздуха и оптимизация аэродинамических характеристик остается одной из ключевых задач для конструкторов в автомобильной промышленности. Это больше не является исключительной прерогативой инженеров гоночных автомобилей и оказывает реально ощутимое влияние на различные аспекты характеристик автомобилей потребительского класса, включая расход топлива, комфорт пассажиров и многое другое. Однако нельзя отрицать, что появление инструментов виртуального прототипирования и моделирования значительно упростило решение этой проблемы, чем раньше. Если вы хотите узнать больше о том, как свести к минимуму сопротивление воздуха с помощью инженерного моделирования, запустите бесплатную пробную версию SimScale и используйте возможности платформы CFD для оптимизации собственных проектов.
Чтобы узнать, как использовать SimScale, посмотрите вебинар «Как оптимизировать гоночные автомобили с помощью облачной CFD», организованный совместно с журналом Racecar Engineering. Просто заполните эту короткую форму, и она будет воспроизводиться автоматически.
Взгляд на самые аэродинамические автомобили из когда-либо созданных
Бензиновые, дизельные или электрические автопроизводители прилагают все усилия, чтобы выжать из своих автомобилей все до последней капли. Большая часть этих усилий направлена на оптимизацию аэродинамики. Уменьшение лобового сопротивления является основным направлением деятельности инженеров, работающих над новейшими высокоэффективными моделями, и за эти годы породило множество инновационных разработок. Мы рассмотрим, почему снижение лобового сопротивления так важно, а также некоторые уникальные транспортные средства, появившиеся в результате этих усилий по оптимизации.
Boo To Air Resistance
График, показывающий увеличение аэродинамического сопротивления и сопротивления качению при увеличении скорости.
Обратите внимание на гораздо более высокий вклад аэродинамического сопротивления, особенно на скоростях шоссе.Независимо от того, ищете ли вы более низкую экономию топлива или просто пытаетесь проехать как можно больше миль от вашей батареи, дрейф — ваш враг. Толкание автомобиля по воздуху требует работы, и чем быстрее вы едете, тем сильнее отталкивает воздух. К сожалению, сопротивление пропорционально квадрату скорости, поэтому при удвоении скорости сила сопротивления увеличивается в четыре раза. При скорости выше примерно 20 км/ч (12,4 мили в час) аэродинамическое сопротивление является самой большой силой, воздействующей на автомобиль, затмевая сопротивление качению по мере увеличения скорости.
Разумеется, можно принять меры для уменьшения этого сопротивления. Помогает создание автомобиля с более гладким профилем, который деликатно разделяет воздух спереди и позволяет ему мягко воссоединяться сзади. Помогает уменьшение размера и количества выступов, а также уменьшение общей лобовой площади автомобиля.
При внимательном отношении к этим факторам автопроизводители могут значительно снизить лобовое сопротивление с сопутствующим повышением эффективности.
О скользкости автомобиля часто говорят с точки зрения коэффициента аэродинамического сопротивления или С д . Это безразмерный коэффициент, который количественно определяет величину сопротивления, создаваемого данным объектом при прохождении через жидкость, такую как вода или воздух. В некоторых анализах также важно учитывать C d A — коэффициент аэродинамического сопротивления, умноженный на лобовую площадь автомобиля. Два автомобиля могут иметь одинаковую обтекаемую форму, но если один больше другого, он, естественно, будет испытывать большее сопротивление.
Коэффициенты сопротивления различных основных форм. Обратите внимание, что важно то, как воздух возвращается вокруг объекта, а не только фронтальный профиль. В качестве ориентира, плоская пластина, пытающаяся проложить себе путь через воздух, выдаст C d 1,28, в то время как пуля с дозвуковой скоростью может пройти 0,295.
Типичные современные седаны и купе имеют коэффициент аэродинамического сопротивления от 0,25 до 0,3, а внедорожники часто имеют более высокие значения от 0,35 до 0,45 из-за их более высокой и более квадратной конструкции. Спорткары, построенные с упором на прижимную силу, естественно, имеют более высокие значения C d из-за индуктивного сопротивления аэродинамических элементов.
1999 Honda Insight, ранний гибридный автомобиль, оказался в нижней части этого диапазона, заявив, что C d равен 0,25, что в то время считалось лучшим в своем классе. Однако новые конкуренты в этом пространстве еще больше улучшили это. Mercedez Benz S 350 BlueTec получил 0,24, как и Tesla Model S при запуске в 2012 году. утверждая цифру всего 0,208. Однако последний Mercedes EQS превосходит оба пункта с показателем всего 0,2.
Дело не только в том, как вы выглядите
Интересно отметить, что в то время как ранние гибриды 1990-х годов имели явно стремительный и обтекаемый дизайн, современные автомобили превзошли эти цифры, не прибегая к таким бесформенным яйцевидным формам.
Часто аэродинамические выгоды можно получить, тщательно формируя поток тонкими способами, а не сосредотачиваясь на макроформе автомобиля в целом. Другие выгоды могут быть получены благодаря техническому прогрессу; Электромобили избавились от больших радиаторов спереди и, таким образом, имеют гораздо более обтекаемые бамперы.
Даже кажущиеся незначительными правила, такие как расстояние от фары до земли или высота капота, могут оказать серьезное влияние на дизайн. Ожидания клиентов в отношении комфорта салона и места для багажа также могут быть проблемой. Таким образом, некоторые из самых низких показателей лобового сопротивления были получены от экспериментальных концептуальных автомобилей. General Motors EV1 1996 года отличается невероятно низким значением C d , составляющим всего 0,19. Это была попытка GM создать настоящий, пригодный для массового использования электромобиль. Автомобиль привлек ярых фанатов среди участников программы ограниченной аренды, но его ущемляли ограниченный запас хода и двухместный салон. Автомобили были отозваны по истечении срока аренды, и подавляющее большинство из них было раздавлено. Точно так же Volkswagen XL1 соответствовал EV1 C d с показателем 0,19 после выпуска в 2013 году. Он был разработан в соответствии с жестким заданием председателя правления Фредерика Пиеха, чтобы выжать 100 км из одного литра дизельного топлива.
Серийная версия, оснащенная двухцилиндровым двигателем мощностью 35 кВт и электродвигателем мощностью 20 кВт, развивала 0,9л/100км в реальных условиях. Ограниченный производственным циклом всего в 200 единиц, автомобиль не имел боковых зеркал или зеркал заднего вида, а также заднего ветрового стекла. Пассажиры сидят тандемом, один за другим, а не бок о бок, чтобы минимизировать фронтальную площадь для максимальной эффективности. Более того, транспортные средства, подобные тем, которые участвовали в World Solar Challenge, разработаны для оптимальной производительности, позволяющей максимально использовать их ограниченную солнечную энергию. Запись Sunraycer 1987 года отличалась обтекаемым кузовом с изображением C 9.0125 d всего 0,125, что вынуждает водителя лежать в машине почти на спине. Аналогичным образом, участие в Эко-марафоне Shell следует примерно той же философии: Eco-runner 8 2018 года набирает скользкие 0,045.
1930-е годы были прекрасным временем для этого парящего взгляда
Schlörwagen (также известный как «геттингерское яйцо») был проектной концепцией, намного опередившей свое время, основанной на шасси Mercedes с задним расположением двигателя и построенной в 1930-х годах.
Однако история оптимизации автомобилей началась намного раньше, чем после 19-го века.Топливный кризис 73 года привел к тому, что американцы начали массово покупать компактные автомобили. Основные аэродинамические концепции, заставляющие объекты скользить по воздуху, применялись давно, когда повальное увлечение модернизацией 1930-х годов коснулось всего, от поездов до автомобилей и тостеров. Tatra T77A был одним из первых автомобилей, разработанных с упором на аэродинамику, но были реализованы и более совершенные конструкции.
Пожалуй, самым экстремальным дизайном того времени был автомобиль, известный как Schlörwagen, названный в честь его конструктора Карла Шлера. Прототип, построенный на шасси Mercedes с задним расположением двигателя, как сообщается, разместил C 9.0125 д 0,15. Это было достигнуто за счет выбора дизайна, который в то время считался диким; весь автомобиль имел форму единого гладкого яйца с минимальными выступами, за что получил прозвище «геттингенское яйцо».
Он полностью закрывал не только задние колеса, но и передние, что требовало кузова шириной 2,10 м, что для того времени считалось смехотворно большим. Окна установлены максимально заподлицо, чтобы уменьшить любые помехи для воздуха, придавая автомобилю футуристический вид, намного опередивший свое время. Однако серийный выпуск автомобиля всерьез не рассматривался, несмотря на впечатляющий дизайн.
В целом, мы, вероятно, увидим, что будущие модели крупных автопроизводителей продолжат тенденцию к снижению показателей лобового сопротивления, поскольку битва за пробег в пространстве электромобилей накаляется. Многие выгоды все еще остаются на столе, поскольку регулирующие органы медленно продвигают правила, касающиеся зеркал и других технологий, которые могут еще больше улучшить цифры. При этом потребители будут по-прежнему требовать минимальных стандартов комфорта, пространства и безопасности, а это означает, что в ближайшее время мы вряд ли будем ездить в заостренных слезинках.
аэродинамика — Является ли коэффициент аэродинамического сопротивления/скорость транспортного средства основанным на лобовой площади?
$\begingroup$
Определяет ли площадь лобовой части автомобиля коэффициент скорости/лобового сопротивления автомобиля?
У Ferrari 458 Cd 0,33, а у Corvette C6 Cd 0,29.
Означает ли это, что у Corvette будет меньше лобовая площадь и больше скорость, а у Ferrari должна быть больше прижимной силы?
- аэродинамика
- сопротивление
$\endgroup$ 92A}$
где:
$F_d$ — сила сопротивления, которая по определению является составляющей силы в направлении скорости потока;
$\rho$ — массовая плотность жидкости;
$u$ — скорость потока объекта относительно жидкости;
$A$ — это область отсчета .
Ответ на ваш первый вопрос: » Да, эталонной площадью для автомобилей и многих других объектов является проекция лобовой площади транспортного средства «. Однако коэффициент сопротивления обычно определяется экспериментально, и известные экспериментальные результаты показывают, что он также зависит от формы объектов, как показано ниже, при условии, что эталонная площадь постоянна для всех случаев:
Должно быть отметил, что «форма» также влияет на «силу сопротивления», в свою очередь, на «тягу вниз», поскольку $F_d = f(C_d, \rho, u, A)$ и $u = f(C_d, \rho , A, F_d)$ Таким образом, из-за сложных взаимозависимостей нет простого ответа на ваши последние вопросы.
0011
$\endgroup$
$\begingroup$
Нет Cd не имеет прямого отношения к лобовой области, он связан с тем, насколько эта лобовая область, движущаяся против потока воздуха, должна бороться с сопротивлением.
Например, рассмотрим прицепы старых полуприцепов. Поскольку они были квадратными, у них был большой Cd и требовалось много топлива даже для небольшой лобовой площади.
В современных прицепах они имеют закругленные углы и регулируемые юбки прицепа на конце, чтобы сделать их более обтекаемыми.
Таким образом, современный прицеп с большей передней площадью имеет меньший Cd, создает меньшее лобовое сопротивление и намного более экономичен.
Если мы возьмем два одинаковых кубика и сравним их, они будут иметь одинаковые Cd. Теперь, если вы добавите полусферу к передней и задней части одного из кубов, его Cd упадет почти до половины другого куба.
$\endgroup$
$\begingroup$
$C_D$ — это коэффициент удобства, используемый инженерами для упрощения гидромеханики автомобилей и аэродинамики.
Упрощенное объяснение состоит в том, что $C_D$ нормализует аэродинамическое сопротивление к лобовой области , чтобы было легче сравнивать разные транспортные средства.
Заявление
«Определяет ли лобовая площадь транспортного средства коэффициент скорость/лобовое сопротивление автомобиля?»
для меня означает, что размер лобной области влияет на $C_D$ (это означает, что чем больше площадь, тем больше $C_D$ или наоборот. Однако $C_D$ в значительной степени не зависит от лобной площади, хотя он рассчитывается из него.92 A}$$
где:
- Аэродинамическая сила $Drag$
- $\rho$ плотность воздуха
- $u$ это скорость
- $A$ — лобовая область.
Так, например, изменение углов заднего спойлера практически не влияет на переднюю часть, но может значительно изменить $C_D$. В этом отношении эти двое независимы.
Единственный случай, о котором я могу думать (возможно, есть и другие), когда $C_D$ зависит от лобовой площади, это когда лобная площадь становится слишком маленькой, и поэтому число Рейнольдса значительно изменяется.
